Pulmonary ROSH Review

Réussis tes devoirs et examens dès maintenant avec Quizwiz!

Which of the following is true regarding active tuberculosis? A cavitary lesion on CT of the chest is pathognomonic Isoniazid treatment for six months is adequate therapy It may have varied appearance on chest X-ray Patients with active tuberculosis need droplet precautions

.The classic X-ray finding on chest radiograph is a cavitary lesion in the upper lobe of the lung. However, tuberculosis can cause varied abnormalites on the X-ray including infiltrate in any portion of the lung. Lymphadenopathy is commonly seen in the hilum on X-ray. Cough is the most common symptom of pulmonary tuberculosis. Additionally, patients may develop fever (more common in the afternoon or evening), night sweats and hemoptysis. Patients with active tuberculosis need airborne precautions, not droplet precautions (D). The size of the droplets is quite small necessitating airborne precautions in a negative pressure room with recirculating air. A cavitary lesion on CT of the chest (A) is not pathognomonic for tuberculosis. There are multiple other infectious causes of cavitary lesions in the chest including Staph aureus, Klebsiella pneumoniae, Pseudomonas aeruginosa, and fungal lesions.

Question: What is the prognosis for a patient with dilated cardiomyopathy?

Approximately 1/3 worsen and die, 1/3 recover completely, and 1/3 have residual cardiac dysfunction.

One Step Further Question: What is Caplan's syndrome?

A patient, typically a miner, with rheumatoid arthritis who acquires any of the pneumoconioses.

One Step Further Question: A completely occluding foreign body will, over time, cause what finding on chest radiograph?

Answer: Atelectasis.

Question: What is the gold standard diagnostic test for spontaneous pneumomediastinum?

The CT scan. Pneumomediastinum Spontaneous, trauma, Valsalva (crack users), Boerhaave syndrome (hx of vomiting) PE: crepitus, Hamman sign CXR: ring around right pulmonary artery Usually self-resolving

Question: What is the most common complication of pneumonia in children?

Dehydration.

One Step Further Question: What histopathological biopsy finding is typically seen in patients with sarcoidosis?

Noncaseating granulomas.

Question: What are the classic symptoms of active tuberculosis?

Persistent malaise, anorexia, weight loss, fever, night sweats, and a chronic cough are classic symptoms

Question: What qualifies as a complicated pneumonia?

Pneumonia involving a parapneumonic effusion or empyema, necrotizing pneumonia, lung abscess, or pneumatocele. Community-acquired Pneumonia (CAP) Patient will be complaining of sudden onset of cough, fatigue, and fever PE will show rales with auscultation of lung fields Most commonly caused by Streptococcus pneumoniae Treatment: macrolides 1st line for children > 5 yrs

Question: What are two clinical situations that can lead to a false-negative D-dimer?

Recent anticoagulation and subacute thrombosis (>7 days). Pulmonary Embolism 95% arise from deep leg veins Sudden onset of symptoms in 50% SOB, CP, tachypnea ECG: sinus tachycardia, nonspecific ST-T changes, right heart strain, S1Q3T3 (classic finding) CXR: nonspecific abnormalities, Hampton's hump (pleural-based wedge infarct), Westermark's sign (vascular cut-off sign) V/Q scan: usually nondiagnostic Low clinical suspicion: negative D-dimer excludes PE Dx of choice: CTPA Rx: anticoagulation, thrombolytics (if massive and HD unstable or submassive with shock, respiratory failure or evidence of moderate to severe RV strain), embolectomy (last resort)

Question: What are the classic ECG changes in a patient with cor pulmonale?

Right axis deviation, R/S ratio >1 in V1 and <1 in V6, P-pulmonale (increased P wave amplitude in leads 2, 3 and aVF). Cor Pulmonale Pulmonary HTN + RVH → right heart failure MC chronic cause: COPD MC acute cause: PE Right heart catheterization

Question: What is the most common bacterial cause of pneumonia in hospitalized patients?

S. pneumoniae.

Question: What is a potential neurologic complication of pertussis infection?

Seizures. Pertussis (Whooping Cough): Patient with a history of nasal congestion, cough, and low-grade fever Complaining of "rapid fire," repetitive coughing followed by an inspiratory "whoop" and post-tussive emesis Most commonly caused by Bordetella pertussis Treatment is a macrolide - azithromycin

Question: What diagnostic criteria is used in Light's criteria?

Serum protein and pleural fluid lactate dehydrogenase are used in Light's criteria, which is used to distinguish between transudative and exudative pleural effusions.

Question: What pathogen causes pneumonia that is associated with bullous myringitis?

Streptococcus pneumoniae. Bullous myringitis was previously linked to Mycoplasma pneumoniae but it appears, based on middle ear aspirate culture results, that typical acute otitis media pathogens are the true cause. Post-viral Pneumonia Patient with a history of influenza Complaining of fever, productive cough with bloody sputum and shortness of breath CXR will show multiple cavitary lesions Most commonly caused by Staphylococcus aureus

Question: A pleural effusion is most difficult to detect in which radiographic position?

Supine.

Question: Can amiodarone be used in patients with heart failure?

Yes, one of the attractive features of using amiodarone over other antidysrhythmics.

What is the most common causative organism in community acquired pneumonia? Klebsiella pneumoniae Mycoplasma pneumoniae Staphylococcus aureus Streptococcus pneumoniae

orrect Answer ( D ) Explanation: Streptococcus pneumoniae is a gram positive cocci and is the most common organism causing community acquired pneumonia. Symptoms of community-acquired pneumonia include fever, tachycardia, chills, shortness of breath, cough, myalgias, and pleuritic chest pain. Some patients may also present with GI symptoms such as nausea, vomiting or diarrhea. Physical exam findings may include increased respiratory rate, crackles, increased tactile fremitus and dullness on percussion.

Question: What type of precautions are necessary for a patient with active/suspected tuberculosis?

Airborne precautions.

Question: What is the significance of a high amylase level in a pleural fluid sample?

A high amylase level usually indicates the presence of pancreatitis, esophageal rupture or malignancy. Pleural Effusion Transudate: CHF (most common) Exudate: infection > malignancy, PE ↓ Breath sounds + dull percussion + ↓ tactile fremitus CXR: blunting of the costophrenic angle

One Step Further Question: What medication is recommended for acute exacerbations of idiopathic pulmonary fibrosis?

Answer: Corticosteroids.

One Step Further Question: What is the patient's forced expiratory volume (FEV1) in a moderate to severe COPD exacerbation versus a mild exacerbation?

Answer: FEV1 is less than 50 percent in moderate to severe exacerbations and greater than 50 percent in mild cases.

Question: What is the most common cause of lung abscess?

Aspiration pneumonia. Lung Abscess Patient will be complaining of several weeks of cough, fever, pleuritic chest pain, weight loss, and night sweats CXR will show area of dense consolidation with an air-fluid level inside a thick-walled cavitary lesion Most commonly caused by aspiration pneumonia Treatment is clindamycin

Question: In which population is Klebsiella pneumonia most commonly seen in?

COPD, Alcoholics and the elderly.

What is the most common cause of minor hemoptysis in the Emergency Department? Acute bronchitis Pneumonia Pulmonary embolism Tuberculosis

Correct Answer ( A ) Explanation: Acute bronchitis is the most common cause of minor hemoptysis in the Emergency Department. Hemoptysis describes the presence of blood in sputum expectorated from the respiratory tract. The majority of episodes of hemoptysis are minor episodes with small amounts of blood. Less than 5% of patients with hemoptysis have life-threatening hemorrhage. Bronchitis results from inflammation in the bronchial tree and can lead to hemoptysis. Additionally, forceful coughing further irritates the inflamed mucosal surfaces and can result in bleeding. Hemoptysis in bronchitis is typically self-limited. Acute Bronchitis Viruses > bacteria Most common cause of minor hemoptysis Hallmark: cough (usually productive), <1week Symptomatic treatment, bronchodilators Routine ABX therapy not indicated

A patient with acute pancreatitis is noted to have a pleural effusion on chest radiography. Which of the following findings would you expect to find on pleural fluid analysis? Elevated amylase concentrations Elevated triglyceride concentrations Low glucose concentrations Low pH of pleural fluid

Correct Answer ( A ) Explanation: Elevated amylase concentrations would be expected on pleural fluid analysis in the setting of acute pancreatitis. Although not routinely tested in pleural fluid samples, amylase measurements can assist when a pleural effusion is caused by a pancreatic or esophageal etiology. The finding of an amylase-rich pleural effusion, defined as either a pleural fluid amylase greater than the upper limits of normal for serum amylase or a pleural fluid to serum amylase ratio greater than 1.0, narrows the differential diagnosis of an exudative effusion to the following major possibilities: acute pancreatitis, chronic pancreatic pleural effusion, esophageal rupture and malignancy. Acute Pancreatitis Patient will be complaining of epigastric pain radiating to the back, nausea, and vomiting PE will show ecchymosis of left flank (GreyTurner sign), umbilical ecchymosis (Cullen sign) Labs will show elevated lipase (best) and amalyse Diagnosis is made by US and Ranson's criteria Most commonly caused by gallstones > alcohol Treatment is IV fluids

65-year-old undomiciled man presents to your emergency department with complaints of fever, productive cough, and shortness of breath. He has a 40-pack-year history of smoking and daily alcohol consumption. His chest radiograph demonstrates a right upper lobe lung infiltrate with an air-fluid level. What is the most likely etiology for this finding? Klebsiella pneumoniae Legionella pneumophilia Mycoplasma pneumoniae Streptococcous pneumoniae

Correct Answer ( A ) Explanation: Klebsiella pneumoniae is a gram-negative encapsulated organism. It occurs most commonly in alcoholic or chronically debilitated patients. Patients with klebsiella pneumonia commonly present with shaking chills, cyanosis, pleuritic chest pain, and a productive cough with characteristic currant-jelly sputum. Chest radiography reveals an infiltrate that is often in the upper lobes (most commonly the right) and is associated with a bulging fissure. If untreated, the infiltrate will progress into a necrotizing lesion with air-fluid levels, and can ultimately lead to development of an empyema. Legionella pneumophila (B) is the most common causative organism of Legionnaire's disease. Legionella is a gram-negative bacillus that lives in the water supply. Transmission occurs through inhalation of contaminated aerosolized droplets emanating from equipment such as cooling towers, evaporative condensers, and shower heads. Legionnaire's disease tends to occur in clusters, with common exposure to the same contaminated source. Spread does not occur from person to person. Infection is associated with hyponatremia, elevated liver enzymes, GI symptoms (watery diarrhea, nausea, vomiting, and abdominal pain), and neurologic signs (seizures, altered mental status, and gait disturbances). Approximately half of patients with Legionnaire's disease will have relative bradycardia. Mycoplasma pneumoniae (C) is considered one of the atypical bacterial causes of pneumonia. It is often referred to as "walking pneumonia," where the patient tends to look better than you would expect, given the diffuse interstitial pattern commonly seen on chest X-ray. Mycoplasma pneumonia is associated with extrapulmonary manifestations such as conjunctivitis, pharyngitis, rash, and pericarditis. Staph aureus pneumonia often affects IV drug abusers, nursing home patients, and those recovering from an influenza infection. Spread to the lungs occurs hematogenously. Chest radiograph reveals a patchy infiltrate that is initially multicentric or peripheral and ultimately progresses to lobar consolidation and abscess formation. Overall, Streptococcus pneumonia (D) remains the most common cause of community-acquired pneumonia. Patients often look ill and tend to have abrupt onset of symptoms, including rigors, before progressing to cough with rust-colored sputum. The chest radiograph of these patients tends to have a single lobar infiltrate, often in the lower lobes of the lun

A 48-year-old man with a history of HIV presents to your office with complaints of fever, cough and shortness of breath. Laboratory testing reveals a CD4 count of 130 cells/microL. Which of the following findings is most likely to be seen on chest X-ray? Ground glass opacification Hampton's hump Honeycombing Kerley B lines

Correct Answer ( A ) Explanation: Pneumocystis jirovecii pneumonia (PCP), previously called Pneumocystis carinii pneumonia, is an opportunistic infection found in individuals infected with HIV who have CD4 counts of less than 200 cells/microL. Symptoms of PCP develop gradually, with fever, cough and shortness of breath being the most common complaints. Patients may also present with fatigue, weight loss, chest pain and chills. Chest X-ray will reveal bilateral, fine, diffuse opacification, often with a ground glass appearance. Patients with HIV whose CD4 counts drop below 200 cells/microL should be given prophylaxis with trimethoprim-sulfamethoxazole and treatment of the infection is with the same agent. Kerley B lines (D) are caused by a thickening of the subpleural interstitium and are often seen in patients with pulmonary edema. Hampton's hump (B) is a wedge of airspace opacity on the periphery of the lung and is seen with pulmonary embolism. Honeycombing (C) is found with end-stage interstitial lung disease such as pneumoconiosis.

An eight-year-old boy comes in your clinic for follow-up. Two days ago, he was diagnosed with pneumonia and started on amoxicillin-clavulanic acid. However, fever and cough persisted and is accompanied by decreased appetite, chest pain, and difficulty breathing. You suspect a complicated pneumonia. Which of the following physical exam findings is consistent with pleural effusion? Dullness to percussion Early inspiratory crackles Trachea deviates away from affected side Vesicular breath sounds

Correct Answer ( A ) Explanation: The boy has history of pneumonia that failed treatment and developed a paraneumonic effusion. Bacterial pneumonias can be associated with complications involving the respiratory tract, which include pleural effusion, empyema, pneumatocoeles, necrotizing pneumonia, and lung abscesses. The incidence of paraneumonic effusions appears to be increasing in the United States. The causative organisms have changed over time with Streptococcus pneumoniae as the predominant organism. In addition, community-acquired Staphylococcus aureus is also becoming more common. The presentation of pleural effusion or empyema may depend upon when the child presents for medical attention. Some children present with symptoms related to empyema while others have been seen earlier in the course and appropriately treated for pneumonia but fail to respond. On physical examination, children may appear ill or occasionally be toxic-appearing. The majority are tachypneic, with fever and cough present in approximately 90 percent of patients. Chest examination may reveal a small degree of new scoliosis that is related to the patient's splinting toward the affected side. There may also be dullness to percussion, decreased air exchange, and possibly a pleural rub, on the side of the fluid collection. Early inspiratory crackles (B) is an exam finding that suggests small airway disease like bronchiolitis. Trachea deviating away from affected side (C) is a finding seen in tension pneumothorax. Vesicular breath sounds (D) is a normal physical examination finding.

A two-year-old boy who is new to your practice is brought to the clinic because of coughing. For the past two weeks, he has had rhinorrhea and coughing. However, the coughing has become worse with more frequent coughing fits and episodes of vomiting. You review his previous records and note that the parents have refused immunizations. On physical examination, his temperature is 38 degrees Celsius, respiratory rate is 35, and lungs are clear to auscultation. During the examination, you note that the boy has a long series of coughs followed by a whoop. Which of the following is the most likely etiologic agent causing the illness? Bordetella pertussis Human metapneumovirus Mycoplasma pneumoniae Respiratory syncytial virus

Correct Answer ( A ) Explanation: The boy has signs and symptoms consistent with Bordetella pertussis infection or simply known as pertussis. The classic presentation of pertussis includes paroxysms of coughing, an inspiratory whoop, and posttussive vomiting. The classic presentation typically occurs as a primary infection in unvaccinated children younger than 10 years of age. There are three stages with the first stage being the catarrhal stage. In this stage the patient develops mild cough and coryza. This may last from one to two weeks. In the paroxysmal stage, coughing spells increase in severity. The paroxysmal cough is distinctive: a long series of coughs between which there is little or no inspiratory effort. The child may gag, develop cyanosis, and appear to be struggling for breath. Then the whoop, or noise made by the forced inspiratory effort that follows the coughing attack. The last stage is the convalescent stage wherein the cough subsides over several weeks to months. Infants older than four months of age and children with suspected pertussis should be treated. Laboratory confirmation should not delay the initiation of treatment.

A 47-year-old woman, with no past medical history and no hospitalizations, presents with cough, green sputum, and fever. Her vitals are T 100.7°F, HR 94, BP 123/76, RR 18, oxygen saturation 97%. She is well appearing and her blood work (CBC and BMP) is unremarkable. A chest X-ray shows a left lower lobe infiltrate. Which of the following represents the best management for this patient? Discharge home with oral antibiotics and follow up Draw blood cultures and discharge home on oral antibiotics Order a chest CT scan Start IV antibiotics, draw blood cultures, and admit

Correct Answer ( A ) Explanation: This otherwise healthy, well appearing patient presents with community-acquired pneumonia (CAP), which affects 2-4 million US patients every year. This patient can be managed as an outpatient with oral antibiotics and follow up. There are a number of prognostic tools for pneumonia to help guide disposition. Among these are the Pneumonia Severity Index (PSI) and CURB-65 both of which would classify this patient as low risk.

What is the most common symptom seen in acute bronchitis? Chest pain Cough Fever Sputum

Correct Answer ( B ) Explanation: Bronchitis is defined as inflammation of the lower airways and is typically caused by viral pathogens. The most common presenting complaint is cough in these patients. In healthy adult patients, acute bronchitis is typically caused by a viral pathogen. As such, it is not amenable to antibiotic therapy. In patients with chronic bronchitis or chronic obstructive pulmonary disease (COPD), exacerbations of symptoms may be the result of bacterial pathogens causing acute bronchitis on top of their chronic bronchitis. Treatment with antibiotics should be considered in this subset of patients with bronchitis. Chest pain (A) often occurs as a result of coughing in patients with bronchitis. Fever (C) may or may not be present and does not indicate a higher likelihood of bacterial infection. Sputum (D) is present in about half of patients with acute bronchitis.

A 4-year-old is brought to the emergency department by his parents after they found him in the backyard shed choking and gagging. Before this event, he was otherwise healthy. His temperature is 101°F, heart rate is 95, blood pressure is 100/60, and respiratory rate 40. On exam, he appears sleepy but continues to cough. Pulmonary findings include moderate retractions and diffuse wheezes. A chest X-ray shows patchy infiltrates. What is the most likely diagnosis? Foreign body aspiration Hydrocarbon ingestion Organophosphate ingestion Status asthmaticus

Correct Answer ( B ) Explanation: Hydrocarbons (e.g. lamp oil, gasoline, lighter fluid, turpentine, benzene, kerosene) can be aspirated easily when ingested and may cause pneumonitis with volumes as low as < 1 mL. The lower viscosity compounds distribute across a larger lung surface. Symptoms of aspiration are non-specific and include grunting, gagging, choking, tachypnea, fever, retractions and persistent coughing. These findings may be delayed, so asymptomatic children should be observed for several hours. The chest X-ray typically shows diffuse bilateral infiltrates. Airway support is important because symptoms can evolve into respiratory failure. Hydrocarbons cause acute respiratory distress syndrome (ARDS) by inactivation of type II pneumocytes, which leads to surfactant deficiency. Foreign body aspiration (A) can cause similar respiratory symptoms, but the chest X-ray will either be normal or show a foreign body if it is radio-opaque. Organophosphate ingestion (C) can cause bronchospasm, but symptoms are not limited to the respiratory tract and include diarrhea, urination, miosis, emesis, lacrimation and salivation. An individual with status asthmaticus can have similar respiratory symptoms, but the fever and chest X-ray findings are inconsistent with the disease process. This patient has no history of wheezing, and status asthmaticus (D) generally has a more insidious onset. During an asthma exacerbation, the chest X-ray may show hyperinflation of the lungs, not patchy infiltrates.

What type of cancer is classically associated with asbestos exposure? Large cell carcinoma Mesothelioma Small cell carcinoma Squamous cell carcinoma

Correct Answer ( B ) Explanation: Mesothelioma is a rare form of cancer that develops from cells of the mesothelium. Mesothelioma is most commonly caused by exposure to asbestos. The most common anatomical site for mesothelioma is the pleura, but it can also arise in the peritoneum, the pericardium, or the tunica vaginalis. Most people who develop mesothelioma have worked in jobs where they inhaled or ingested asbestos fibers, or were exposed to airborne asbestos dust and fibers in other ways. Concomitant cigarette smoking has been shown to accelerate the progression of asbestosis and increase cancer risk further. Early changes associated with asbestosis on chest radiograph include bilateral, irregular, linear opacities at the lung periphery, especially pronounced in the lower lung fields. The middle and upper lobes are typically involved later in the course of the disease. Mesothelioma commonly metastasizes, but death in these patients is usually due to local invasion.

In a patient with suspected HIV infection and shortness of breath, which of the following findings is most suggestive of Pneumocystis jiroveci pneumonia? Bradycardia despite relative volume depletion Elevated serum lactate dehydrogenase Hyponatremia Unilateral lobar consolidation on chest radiograph

Correct Answer ( B ) Explanation: Pneumocystis pneumonia is one of the most common opportunistic infections in patients with HIV. Patients typically present with an insidious nonproductive cough, dyspnea, unexplained fever for longer than two weeks, chest pain, and fatigue. The greater the elevation of lactate dehydrogenase (LDH), the worse the prognosis. Relative bradycardia (A) is commonly associated with typhoid fever (Salmonella typhi), Legionnaire's disease (Legionella pneumophila), infectious mononucleosis, and pneumonia caused by Chlamydia species. The syndrome of inappropriate antidiuretic hormone secretion (SIADH) is one of several causes of hyponatremia (C). SIADH may be seen in patients with certain pulmonary infections, including Legionnaire's disease and tuberculosis

Which of the following is the first line treatment for latent TB infection? Doxycycline Isoniazid Lamivudine/Zidovudine Penicillin

Correct Answer ( B ) Explanation: Standard therapy for latent TB infection (LTBI) is isoniazid (INH). In the United States, latent tuberculosis infection is the most prevalent form of tuberculosis. LTBI is the term given to patients with a positive purified protein derivative (PPD) skin test without evidence of active TB. PPD has been used for more than 100 years and relies on delayed-type hypersensitivity (DTH) to M. tuberculosis cellular proteins. Because PPD relies on DTH, any factor that reduces the DTH affects the host response to PPD. The most common clinical example is use of corticosteroids, which blunt the DTH response and can complicate PPD interpretation. Therefore, PPD testing should not be performed while a patient is taking corticosteroids. Also, TB testing should be targeted to those with higher risk of infection and should not routinely be done in those with low risk. Patients at increased risk for progression to active TB include those who have been recently infected (recent PPD converters); patients who are HIV seropositive; patients who have silicosis, diabetes, or chronic renal failure (including those receiving hemodialysis); solid-organ transplant recipients; patients with gastrectomy or jejunoileal bypass or head and neck cancer; injection drug users; patients with chest radiograph evidence of prior TB; and patients who weigh at least 5% less than ideal body weight. Patients taking chronic corticosteroid therapy and those who are to receive tumor necrosis factor alpha (TNF-alpha) blockers (e.g., infliximab) are also at risk.

An 18-year-old woman with a history of asthma presents to her primary care provider for follow-up. Currently she experiences symptoms of coughing, wheezing and shortness of breath 3 days a week and uses her rescue inhaler 3 days a week. She awakens at night with similar symptoms 3 times a month. She reports some minor interference with her daily routine. Which of the following options is an appropriate management plan at this time? AHigh dose inhaled corticosteroid and short-acting beta-agonist as needed BLow dose inhaled corticosteroid and short-acting beta-agonist as needed COral corticosteroid and short-acting beta-agonist as needed DShort acting beta-agonist as needed

Correct Answer ( B ) Explanation: This patient can be classified as having mild persistent asthma. The recommended regimen for her includes a low dose inhaled corticosteroid (ICS) and short-acting beta-agonist (rescue inhaler). Asthma is a type of obstructive lung disease marked by chronic inflammation of the lower airways. Classic symptoms include recurrent episodes of cough, shortness of breath and wheezing. A pulmonary function test is initially obtained to confirm diagnosis. Test results demonstrate a decreased FEV1 (forced expiratory volume in 1 second) to FVC (forced vital capacity). Administration of a short acting beta agonist shows some reversibility. A step-wise approach is used in treating symptoms of asthma based on the severity. Inhaled corticosteroids are useful in reducing airway inflammation and bronchial hyper-responsiveness. Peak flow monitoring is an effective way to assess asthma control at home. In addition, it is important to reduce airway irritants and triggers to avoid exacerbation of symptoms.

72-year-old man with a 40 pack-year smoking history presents to the emergency department complaining of shortness of breath and cough for two days. He reports difficulty breathing when walking short distances in his home. His cough is nonproductive. His oxygen saturation is 87% on room air. He is alert but has difficulty speaking in complete sentences. On auscultation, there is diffuse expiratory wheezing. Chest radiograph shows hyperexpansion and flattened diaphragms. Which of the following is the most appropriate therapy? Administration of supplemental oxygen, intravenous methylprednisolone, theophylline Administration of supplemental oxygen, nebulized albuterol and ipratropium, oral prednisone Administration of supplemental oxygen, nebulized albuterol, intravenous antibiotics Administration of supplemental oxygen, oral prednisone, terbutaline

Correct Answer ( B ) Explanation: The most appropriate therapy in a patient with a chronic obstructive pulmonary disease (COPD) exacerbation is administration of supplemental oxygen, nebulized albuterol and ipratropium, and oral prednisone. Antibiotics are necessary in patients with increased dyspnea, increased sputum production, and increased sputum purulence. Administration of supplemental oxygen should target a pulse oximetry between 88 and 92 percent to avoid worsened hypercapnia. Goal arterial oxygen tension is 60 to 70 mm Hg. A nasal cannula can effectively provide oxygen at flow rates up to 6 liters per minute and is much more convenient and comfortable for the patient, especially when eating. Venturi masks, another method of oxygen delivery, are advantageous in that they deliver precise oxygen content but are not as well tolerated by patients. Endotracheal intubation must be considered if the patient has altered mental status, profound acidemia, or cardiac dysrhythmia. Inhaled short-acting beta agonists such as albuterol are the mainstay in treatment of COPD exacerbations. These medications can be given via nebulizer or metered-dose inhaler with a spacer, though in the setting of an acute exacerbation nebulized therapy is preferred. Short-acting beta agonists are frequently administered with a short-acting anticholinergic such as ipratropium, even though evidence regarding this is conflicted. A short course (less than fourteen days) of an oral glucocorticoid is recommended in patients with a COPD exacerbation if they are tolerating oral intake, are not in shock, and are having a mild to moderate exacerbation. The reason for this is that oral glucocorticoids are more rapidly absorbed, achieving peak levels within one hour of ingestion, and are almost completely bioavailable. In addition to the exceptions noted above, intravenous glucocorticoids may be administered if the patient already failed treatment with oral glucocorticoids. There is debate about the optimal duration of treatment with glucocorticoids. Global Initiative for Chronic Obstructive Lung Disease (GOLD) guidelines recommend the equivalent of forty milligrams of prednisone daily for five days. Other societies suggest a course of less than fourteen days. The duration of treatment is determined by the severity of the exacerbation and the patient's response to treatment. At the end of the glucocorticoid treatment course, the medication can be discontinued rather than tapered. Treatment courses in COPD are generally too short to cause adrenal suppression, so this should not be a concern when stopping the patient's medication. In addition to the above, smoking cessation should be addressed with the patient.

A 68-year-old man presents for a wellness visit. He is a former smoker, having quit 3 years ago. He has a 35 pack-year history of smoking. He has no other comorbidities and is feeling well at the time of his visit. What screening needs to be performed with regard to lung cancer? Chest X-ray Low dose chest CT scan No screening is indicated Pulmonary function tests

Correct Answer ( B ) Explanation: This patient needs annual low dose chest CT scans until he has been a non-smoker for 15 years consecutively. United States Preventive Services Task Force (USPSTF) guidelines state patients aged 55-80 years old who are either smokers or former smokers who quit within the past 15 years and have a total of 30 pack-years or more need annual low dose chest CTs until they have been non-smokers for 15 or more years. Lung cancer is the third most common cancer in the USA and the top cause of cancer related deaths. Smoking is the most common risk factor for lung cancer and is associated with 85% of diagnosed lung cancer in the USA. Incidence increases with age and cumulative smoke exposure. Low dose CT scanning has a sensitivity of 93.8% and a specificity of 73.4% when screening for lung cancer.

A 65-year-old man presents with cough and progressive dyspnea on exertion. He has no significant cardiac disease. He worked as an asbestos remover for thirty years. Examination reveals crackles but no wheezing. Pulmonary imaging reveals multiple basilar round opacities and pleural plaques. This patient is most at risk for developing which of the following diseases? ALoffler's syndrome BLofgren's syndrome CMesothelioma DMultiple myeloma

Correct Answer ( C ) Explanation: Asbestosis is one of the occupational lung diseases known as pneumoconiosis. It is a chronic inflammatory condition of the pulmonary interstitium, resulting in fibrosis and restrictive lung disease. It occurs in those chronically exposed to asbestos, namely manufacturers of asbestos-containing products or those involved in the removal of asbestos-containing materials. Symptoms usually only manifest after long exposure occurs. They include a progressive and sometimes severe dyspnea, especially on exertion, and dry cough. Bibasilar crackles are typical. In addition to the interstitial fibrosis, pleural thickening and plaques, as well as pleural effusion, occurs. Respiratory failure may be the end result of this condition. Pulmonary function testing reveals restrictive ventilatory defects. Diagnosis is mostly based on the classic occupational history and the key imaging findings of pleural plaques and small, basilar opacities. Although not necessary in most cases, biopsy results will show asbestos bodies. A rare form of cancer, malignant mesothelioma, is most commonly caused by chronic asbestos exposure. Although this cancer mainly occurs in the mesothelial cells of the pleura, it can also develop in the pericardium, peritoneum and tunica vaginalis.

A 37-year-old man presents with cough and shortness of breath. Vital signs are T 102°F, BP 110/76, HR 108, RR 20, and oxygen saturation of 92% on room air. His chest X-ray is shown above. Which of the following helps determine the causative organism? Exposure to white powder History of smoking Recent influenza infection Residence in Connecticut

Correct Answer ( C ) Explanation: Cavitary lesions of the lung have multiple causes, including both infectious and non-infectious etiologies. These include bacterial pneumonia, fungal disease, tuberculosis, malignancies and some pulmonary vascular disease. The bacteria most commonly associated with cavitations are anaerobes, aerobic gram-negative bacilli and Staph aureus. After a recent influenza infection, patients may develop a Staph aureus pneumonia. Of particular concern is community-associated methicillin-resistant Staph aureus (CA-MRSA) after influenza especially in a rapidly progressive pneumonia in younger, healthy patients. Staph pneumonias often have necrotizing features creating the cavitation and may also lead to the development of pneumatoceles

A 58-year-old man presents with shortness of breath for 2 days. He complains of a six-month history of a dry cough, unintentional weight loss, and night sweats. He has no past medical history, but he has a 40 pack-year smoking history. In the ED, his vital signs are BP 132/76, HR 72, RR 16, oxygen saturation 96% on room air, and temperature 98.8°F. An ECG reveals no acute abnormality, and a chest X-ray shows a right middle lobe irregular mass and a right-sided pleural effusion. A thoracentesis is performed. What findings would be expected on pleural fluid analysis? Fluid:Blood LDH ratio < 0.6 Fluid:Blood Protein ratio < 0.5 Glucose < 60 mg/dL LDH < 200

Correct Answer ( C ) Explanation: Given his extensive smoking history, irregular mass noted on chest X-ray, and his recent unintentional weight loss, night sweats, and chronic cough, his pleural effusion is most likely secondary to an underlying lung malignancy. A pleural effusion is defined as more than 15 cc of fluid between the parietal and visceral pleuras. The most common causes are congestive heart failure (CHF), pneumonia, and malignancy. An effusion can be classified as transudative or exudative. A transudative effusion occurs secondary to increased hydrostatic pressure or decreased oncotic pressure. Common causes of transudative effusions include CHF, hypoalbuminemia, cirrhosis, and nephrotic syndrome. An exudative effusion is caused by increased permeability of pleural surfaces or decreased lymphatic flow. Common causes of exudative effusions include infection, such as pneumonia or tuberculosis, neoplasm, connective tissue disease, pulmonary embolism, uremia, pancreatitis, or esophageal rupture. A thoracentesis can be performed to analyze the fluid and determine a cause. Light's Criteria defines a transudative effusion by a LDH <200 (or less than 2/3 of the upper limit of the normal serum LDH level), a fluid:blood LDH ratio <0.6, and a fluid:blood protein ratio <0.5. In an exudative effusion, the glucose level is <60 mg/dL. In addition, if an exudative effusion is suspected, other tests such as fluid amylase, cell count, gram stain, culture, and cytology should be obtained. Fluid:blood LDH ratio <0.6 (A), fluid:blood protein ratio <0.5 (B), and LDH<200 (D) are all representative of a transudative effusion.

A 27-gestational-week-old infant is delivered in the emergency department. The neonate is cyanotic, tachypneic and shows chest wall retractions. An emergent chest radiograph reveals a diffuse ground-glass appearance. Meconium aspiration is not suspected. Which of the following is the most likely diagnosis? Erythroblastosis fetalis Hydrops fetalis Infant respiratory distress syndrome Persistent pulmonary hypertension of the newborn

Correct Answer ( C ) Explanation: Infant (neonatal) respiratory distress syndrome is the new term for hyaline membrane disease. It is also known as respiratory distress syndrome of the newborn and surfactant deficiency disorder. It occurs in premature infants whose pulmonary system has not completely matured, namely those born before 34 weeks (especially between 26 and 28 weeks) gestational age, affecting about 1% of all newborns. It is due to a deficiency of surfactant, a lipid-protein substance produced by type II pneumocytes which decreases surface tension in, and prevents collapse of, alveoli. Symptoms include tachypnea, tachycardia, cyanosis, nasal flaring, expiratory grunting and chest wall retractions. Diagnosis is made clinically and radiographically. The typical radiograph shows decreased lung volume, air-bronchograms and a ground-glass appearance. Treatment includes oxygen, continuous positive airway pressure, exogenous surfactant and in severe cases, intubation and extracoporeal membrane oxygenation. Hemolytic disease of the newborn, also known as Rh disease, or Rh incompatibility, occurs in subsequent pregnancies of a Rh negative woman and RH positive fetus. Mild cases present as neonatal anemia and reticulocytosis, while severe causes result in erythroblastosis (A), hyrops fetalis or stillbirth. Hydrops fetalis (B) is an abnormal accumulation of edema in at least two fetal compartments, due to several maternofetal immune and nonimmune conditions. Its common manifestation is prenatal heart failure, not respiratory failure. Persistent pulmonary hypertension of the newborn (PPHN) (D) is due to a failure of the circulatory transition which occurs during birth, mainly due to a right-to-left intracardiac shunt. It occurs mainly in term or near-term neonates, and only very infrequently in preterm neonates. Lung fields are mostly clear in radiographs of those neonates diagnosed with PPHN. It is commonly caused by meconium aspiration.

A transudative pleural effusion is identified after thoracentesis. Which of the following clinical scenarios is most consistent with this type of effusion? 27-year-old woman with a lupus flare 47-year-old woman alcoholic with an elevated lipase 65-year-old man with an ejection fraction of 15% and pulmonary edema 72-year-old man recently diagnosed with lung cancer

Correct Answer ( C ) Explanation: Pleural effusions are broken into two main classes: transudative and exudative. Transudative effusions develop when there is increased hydrostatic pressure or decreased oncotic pressure causing the translocation of fluid. Approximately 90% of transudative effusions are caused by congestive heart failure as in the clinical scenario described Exudative effusions result from an increase in membrane permeability and can be seen in pancreatitis (B), lupus serositis (A) and malignancy (D). The most common cause of exudative effusion is pneumonia when the adjacent pleural tissue becomes inflamed. Malignant effusions are the second most common cause.

A young woman with dyspnea on exertion presents for evaluation. She is subsequently diagnosed with sarcoidosis. Which of the following would you expect to appear in her lymph node biopsy report? Acid-fast organisms Congo red stain birefringence Noncaseating granulomas Reed-Sternberg cells

Correct Answer ( C ) Explanation: Sarcoidosis is an idiopathic widespread inflammatory condition. It is distinguished by accumulation of T cells and mononuclear phagocytes in affected organs, along with noncaseating granulomas and disturbance in normal tissue architecture. The lung is the most commonly involved organ, with skin, eye, liver, and lymph node involvement also common. The disorder can be acute, subacute and self-limiting, or a chronic waxing and waning disease occurring over many years. In the U.S.A., African-Americans are affected 10x more than Caucasians. Patients may be asymptomatic or complain of fatigue, malaise, weight loss, and fever. Commonly, patients present with symptoms based on the involved organ. For the lungs, this may include exertional dyspnea; dry, nonproductive cough; and vague chest pain. Exam findings are usually normal but may sometimes include crackles. Lymphadenopathy may be present. Chest X-ray is classic for bilateral hilar lymphadenopathy. Diagnosis is made by fiberoptic bronchoscopy with transbronchial biopsy showing a sarcoid granuloma, which is a well-formed noncaseating epithelioid cell granuloma surrounded by a rim of fibroblasts and lymphocytes. Management includes systemic corticosteroids as first-line therapy with methotrexate as an alternative. Acid-fast organisms (A), such as Mycobacterium species, are present in pulmonary tissues of patients with tuberculosis, not pulmonary sarcoidosis. In the presence of amyloid fibrils, congo red stain will appear as green birefringence (B) under polarized light. This is a diagnostic test for amyloidosis, not sarcoidosis. Reed-Sternberg cells (D) are found in Hodgkin lymphoma, not sarcoidosis.

A 43-year-old man with asthma presents with wheezing. After 6 inhaled albuterol treatments he feels better and his lungs are clear. Which of the following is true regarding further management? Intravenous corticosteroid is indicated Intravenous magnesium sulfate is indicated Oral corticosteroid is indicated Oral respiratory antibiotic is indicated

Correct Answer ( C ) Explanation: The patient presents with a moderate asthma exacerbation that has resolved with beta-agonists and should have oral corticosteroids added to his treatment. Corticosteroids inhibit the release of inflammatory mediators and cytokines and decrease recruitment of inflammatory cells in asthma. This results in decreased airway inflammation and secondarily limits induced bronchoconstriction. The effect of steroids begins within hours in an acute asthma exacerbation and reduces both the rate of relapse and rate of admission in severe attacks. In general, patients with moderate to severe reactions should have short-course corticosteroids added to their treatment regimens in the Emergency Department. Oral prednisone or prednisolone are appropriate interventions.

62-year-old man with a history of chronic obstructive pulmonary disease presents with cough, headache, dyspnea, and watery diarrhea that started six days ago. He was seen at a local urgent care four days ago and prescribed amoxicillin-clavulanate without improvement. He is ill-appearing with a fever of 38.7°C and inspiratory rales on auscultation. Which of the following results would be most consistent with his diagnosis? Right upper lobe infiltrate with bulging fissure on chest X-ray Serum potassium 6 mEq/L Serum sodium 128 mEq/L Sputum gram stain with gram positive cocci in pairs

Correct Answer ( C ) Explanation: This patient is presenting with Legionella pneumonia. It was first identified in 1976 after an outbreak at the American Legion Convention in Philadelphia. Legionella is a gram-negative facultative intracellular bacillus and is found in aquatic environments. Transmission is from inhalation of contaminated aerosols. It is a more common cause of pneumonia in the summer months when other pathogens are less frequently seen. Patients at risk include smokers, those with underlying respiratory disease (e.g. COPD) or immunosuppression, and men > 50 years of age. Patients present with fevers, malaise, and myalgias. The cough is initially dry, but becomes productive of purulent sputum as the illness progresses. Gastrointestinal symptoms are frequently seen, including diarrhea in about 50% of patients. Hyponatremia is also commonly seen. Typical chest X-ray findings include unilateral patchy alveolar lower lobe infiltrates occasionally accompanied by pleural effusions. Laboratory studies include leukocytosis, elevated liver transaminases, and hyponatremia. Diagnosis is made based on clinical findings and urinary antigen assay. The antibiotic of choice is azithromycin. Alternative agents include trimethoprim-sulfamethoxazole and fluoroquinolones. Upper lobe infiltrates with a bulging fissure (A) are seen in Klebsiella pneumoniae. The gram stain in Legionella typically shows numerous polymorphonuclear lymphocytes (PMNs), but no predominant organism. Sputum gram stain with gram positive cocci in pairs (D) is consistent with pneumococcal pneumonia. Diarrhea typically leads to hypokalemia, not hyperkalemia (B). Patients with Legionella may have hyponatremia, but not hypernatremia.

16-year-old boy is taken to his doctor for snoring. His mother reports that his snoring keeps others in the house awake and that sometimes his breathing pauses during sleep with gasping or choking. His teachers report that he falls asleep frequently at school. On exam, he has a body mass index of 31 kg/m² and has enlarged tonsils. His symptoms are concerning for obstructive sleep apnea so the pediatrician refers him for an overnight polysomnography. Which of the following can be a longterm complication of obstructive sleep apnea? Cerebrovascular accident Lung scarring Nasal polyps Pulmonary hypertension

Correct Answer ( D ) Explanation: Children with obstructive sleep apnea (OSA) often are obese, have enlarged tonsils or have significant allergies. OSA can lead to abnormal growth and development, bedwetting, behavioral and learning problems, daytime sleepiness, and hyperactivity. Long-standing repetitive oxygen desaturations and hypercapnia episodes during sleep can lead to vascular remodeling and pulmonary hypertension which can lead to cor pulmonale (right ventricular hypertrophy). Treatment may include weight loss, managing allergic rhinitis, and removal of the adenoids and tonsils. Nonsurgical approaches to treatment include weight loss, CPAP, and bite guards that bring the lower jaw forward. Surgical treatments include uvulopalatopharyngoplasty, jaw surgery, or removal of the tonsils and adenoids.

A new nursing home employee is being screened for tuberculosis and develops a 6 mm induration on his right volar forearm after injection of a purified protein derivative (PPD). This reading indicates prior tuberculosis exposure if he has had which of the following life circumstances? He is a former guard at a correctional facility He is a HIV-negative injection drug user He is a recent immigrant from Latin America He is an organ recipient on daily immunosuppressives

Correct Answer ( D ) Explanation: An organ recipient on daily immunosuppression therapy is among the group of people who would be considered to have a positive purified protein derivative tuberculin skin test, or PPD, at a reading of 5 mm or greater of induration. Other patients considered to have a positive PPD at 5 mm would include those with recent contact to people with active tuberculosis, those with chest radiographs suggestive of infection, and any HIV-positive patients. Patients who recently emigrated from countries with a high tuberculosis prevalence, are injection drug users, or are residents or employees of high-congregate areas are considered to have a positive PPD reading at 10 mm. Many immunocompromising conditions also allow for a positive reading at 10 mm. Otherwise healthy adults without significant risk of prior exposure would have a positive PPD read at 15 mm of induration. The tuberculin skin test, in which purified protein derivative is placed under the skin of the volar forearm, is the most common method used in screening at-risk people for tuberculosis exposure. This test has high sensitivity and specificity. However, patients who were previously vaccinated with bacillus Calmette-Géurin may falsely test positive. In these patients, an interferon gamma release assay should replace the PPD for tuberculosis screening. All suspected cases of tuberculosis should be reported to government health authorities. A sputum culture should be ordered to determine drug sensitivities of the isolate. Beyond routine screenings, clinicians should be alert for signs and symptoms of pulmonary tuberculosis in patients. These include persistent cough, anorexia, weight loss, fever, night sweats, and blood-streaked sputum production. If tuberculosis is diagnosed, patients must complete treatment with a 6-9 month course of an appropriate anti-tuberculosis drug regimen to prevent the spread of illness. A guard at a correctional facility (A), an HIV-negative injection drug user (B), and a recent immigrant from Latin America (C) would all need a PPD reading of 10 mm of induration or greater for there to be concern of a previous exposure to tuberculosis.

A 68-year-old woman presents with abrupt onset of fever, malaise, body aches, headache, and sore throat. Rapid strep test is negative and nasal smear test is positive for influenza B. Which of the following is the most appropriate therapy? Acyclovir Amantadine Azithromycin Oseltamivir

Correct Answer ( D ) Explanation: Oseltamivir is a neuraminidase inhibitor that is effective against influenza A and B. Influenza is an acute respiratory viral illness, which occurs mainly during the winter in outbreaks and pandemics worldwide. Although it is acutely debilitating, influenza is usually a self-limiting disease. However, it has increased morbidity and mortality in older and very young populations, as well as those with respiratory complications. Transmission of the virus is through respiratory secretions (coughing and sneezing). Symptoms generally include rapid onset of fever and myalgias, often accompanied by cough, sore throat, chills, and headache. Early use of antiviral medication (less than 2 days after onset) is recommended by the CDC for five days in high-risk patients, which results in reduced mortality and morbidity. Acyclovir (A) is used to treat viral infections due to certain herpes viruses, including common cold sores and shingles. Amantadine (B) is effective against influenza A, but ineffective against influenza B.

A 72-year-old man is sent to the interventional radiology department by his primary care doctor to undergo a thoracentesis after a chest radiograph revealed a moderate left-sided pleural effusion. Analysis reveals a pleural fluid/serum protein ratio > 0.9 and a pleural fluid/serum LDH ratio > 0.8. Based on these findings, what is the most likely diagnosis? Cirrhosis with ascites Congestive heart failure Hypoalbuminemia Malignancy Nephrotic syndrome

Correct Answer ( D ) Explanation: Pleural effusions result from the accumulation of fluid between the visceral and parietal pleura. They form as a result of two different processes. Transudative effusions contain little to no protein and result from an increase in hydrostatic pressure or a decrease in oncotic pressure of the pulmonary microvasculature. In contrast, exudative effusions are protein rich. They occur due to increased permeability of the pleura or impaired lymphatic drainage. Light's criteria are used to distinguish transudative from exudative effusions. Out of the answer choices above, malignancy is the only one associated with an exudative effusion. Congestive heart failure (B) is the most common cause of transudative effusions and results from increased hydrostatic pressure. Cirrhosis of the liver (A) and nephrotic syndrome (E) result in transudates due to combined increases in hydrostatic pressure and decreases in oncotic pressure. Hypoalbuminemia (C) can also lead to a transudative effusion due to a significant decrease in oncotic pressure.

A 39-year-old stonemason presents with dyspnea that has been present for several months. A chest X-ray shows an eggshell calcification of the hilar lymph nodes and multiple small 1-2 mm nodules throughout the lung. Which of the following is the most likely diagnosis? Asbestosis Byssinosis Siderosis Silicosis

Correct Answer ( D ) Explanation: Silicosis is one of many forms of pneumoconiosis, a chronic fibrotic lung disease caused by the inhalation of inorganic materials. Silicosis is due to the prolonged inhalation of silica (silicon dioxide) particles. It is most likely in sandblasting, rock mining, and stone cutting. Silicosis is usually asymptomatic, but prolonged-exposure cases result in dyspnea and obstructive, restrictive pulmonary dysfunction. Small, round nodules are formed through the lung. Chest radiography showing calcification of the hilar lymph nodes ("eggshell" calcification) is highly suggestive of silicosis. Treatment is supportive and smoking cessation is essential. Corticosteroids can be used to relieve any chronic alveolitis. Patients with silicosis are at increased risk of pulmonary tuberculosis. If active or inactive tuberculosis is suspected, multi-drug treatment should be initiated. Byssinosis (B) is an asthma-like disorder caused by the inhalation of cotton dust in textile workers. Siderosis (C) is a form of pneumoconiosis caused by inhalation of metallic iron or iron oxide, most likely in the mining, welding, and foundry occupations.

Which of the following is the most common sign seen in patients presenting with a pulmonary embolism? Hemoptysis Hypoxia Swollen calf Tachypnea

Correct Answer ( D ) Explanation: Tachypnea is the most common vital sign abnormality seen in acute pulmonary embolism (PE). PE is a common, life-threatening disease caused by the occlusion of pulmonary arteries by blood clots. The majority of these clots originate in the deep venous system (more commonly in the legs than in the upper body). Patients can present with various symptoms including weakness, shortness of breath, malaise, syncope, dizziness or chest pain. In addition, PE can cause changes to all of the major vital signs. Sinus tachycardia is the most common abnormality found on ECG.

Which of the following is most appropriate in the treatment plan for a patient with idiopathic pulmonary fibrosis? Albuterol Azathioprine Bosentan Pulmonary rehabilitation

Correct Answer ( D ) Explanation: The goal in the treatment of idiopathic pulmonary fibrosis is to reduce symptoms, prevent acute exacerbations, vaccinate, slow progression and increase survival. There are several treatment options, however, most medication trials have shown no benefit. Sufficient clinical evidence that any treatment improves survival or quality of life for patients with idiopathic pulmonary fibrosis (IPF) is lacking. Supportive care, supplemental oxygen and pulmonary rehabilitation appear to be the most beneficial options. Lung transplantation may be an option in those with disease progression and minimal comorbidities. IPF prognosis is poor. Only 20-30% of patients survive five years after diagnosis.

A 42-year-old woman who spent two days hospitalized after she underwent an appendectomy three weeks ago presents with cough, green sputum and fever. Her vitals are T 100.7°F, HR 94, BP 123/76, RR 18, and oxygen saturation 97%. She is well appearing and her blood work (CBC and BMP) is unremarkable. A chest X-ray shows a left lower lobe infiltrate. Which of the following represents the best management for this patient? Discharge home with oral antibiotics and follow up Draw blood cultures and discharge home on oral antibiotics Order a chest CT scan Start IV antibiotics and admit

Correct Answer ( D ) Explanation: This patient has a health-care associated pneumonia (HCAP) requiring IV antibiotics and admission. HCAP is defined as infection occurring within 90 days of a 2-day or longer hospitalization; in a nursing home or long-term care residence; within 30 days of receiving intravenous antibacterial therapy, chemotherapy, or wound care or after a hospital or hemodialysis clinic visit. HCAP requires IV broad spectrum antibiotics because it may involve both the typical pathogens involved in community-acquired pneumonia (CAP) (Mycoplasma pneumonia, Haemophilus influenzae, Streptococcus pneumoniae and Chlamydia pneumoniae) as well as more resistant organisms (Acinetobacter species, Pseudomonas species, Staphylococcus aureus (including MRSA) Enterobacter species, Escherichia coli, Proteus species, Klebsiella species etc.). Treatment should be as follows (one antibiotic from each category):

A 2-month-old boy presents with a fever and cough. Which of the following is suggestive of Chlamydia pneumonia in this infant? Bullous myringitis Diarrhea Rusty-colored sputum Staccato cough

Correct Answer ( D ) Explanation: There are multiple causes of bacterial pneumonia and key characteristics of the patient's clinical picture may help to identify the causative organism. Chlamydia trachomatis pneumonia is associated with the characteristic staccato cough. Pneumonia due to C. trachomatis is recognized in most affected infants between 3 and 16 weeks of age, although essentially all are symptomatic before 8 weeks. Initial therapy for chlamydial pneumonia should be initiated on a presumptive diagnosis, based on clinical and radiographic findings, until diagnostic test results are available.Treatment is with a macrolide antibiotic. The American Academy of Pediatrics (AAP) Committee on Infectious Disease and the Centers for Disease Control and Prevention (CDC) recommend oral azithromycin (10 mg/kg on day one and then 5 mg/kg on day two-four) for either chlamydial pneumonia. For chlamydial conjunctivitis, erythromycin should be used. Both erythromycin and azithromycin are associated with increased risk of infantile hypertrophic pyloric stenosis, particularly in infants younger than two weeks.The infant's mother and her sexual partners should be evaluated and treated for C. trachomatis infection. They also should be evaluated for other sexually transmitted diseases. Bullous myringitis (A) are blisters (bullae) that are seen on the tympanic membrane during inspection of the middle ear. Bullous myringitis was previously linked to Mycoplasma pneumoniae but it appears, based on middle ear aspirate culture results, that typical acute otitis media pathogens (S. pneumoniae) are the most likely pathogens. Diarrhea (B) and gastrointestinal complaints are classically associated with Legionella pneumophila. Rusty-colored (C) or bloody sputum is associated with Streptococcus pneumoniae infection. Patients with Streptococcal infection are often acutely ill appearing and classically describe the onset of a shaking chill followed by fever and cough.

A 50-year-old man with a history of hypertension, diabetes and stage III chronic kidney disease with a GFR of 45, presents to the emergency department complaining of shortness of breath and stabbing chest pain for the past hour. Vital signs are BP 145/70, RR 36, HR 115, and pulse oximetry 89% on room air. An ECG reveals sinus tachycardia. Two days ago he returned to Los Angeles on a business trip from China. Which of the following is the most appropriate diagnostic test for this patient? AChest CT angiography BChest radiography CPulmonary angiography DVentilation/perfusion scan

Correct Answer ( D ) Explanation: Ventilation/perfusion scan is the most appropriate diagnostic imaging modality for this patient who likely has a pulmonary embolism. For most patients with suspected pulmonary embolism, chest CT angiography is the first-choice diagnostic imaging modality because it is sensitive and specific for the diagnosis. However, this patient has stage III chronic kidney disease and chest CT angiography is contraindicated. Ventilation/perfusion scanning is reserved for those with suspected pulmonary embolism in whom chest CT angiography is contraindicated such as renal insufficiency with an estimated GFR < 60, contrast allergy, pregnancy or morbid obesity. In general, ventilation/perfusion scanning is a sensitive test for the diagnosis of pulmonary embolism, but is poorly specific due to the high number of false-positive test results

Question: What is the preferred form of management in patients with sarcoidosis?

Corticosteroids (eg, prednisone).

One Step Further Question: Which antibiotic should be used if macrolide resistance is suspected or documented in a patient with atypical pneumonia?

Doxycycline or a fluoroquinolone.

27-year-old man with a history of asthma presents to your office for his annual exam. He reports that in the past month he has experienced wheezing and shortness of breath about once per week which resulted in using his rescue inhaler. He woke up once because of coughing. He has a peak flow meter and home readings have been 85-90% of his personal best. Which of the following best describes the classification of his asthma? Intermittent Mild persistent Moderate persistent Severe persistent

Explanation: Asthma severity is determined by monitoring patient symptoms over the past 2-4 weeks, determination of the patient's current lung function and number of asthma exacerbations that require oral glucocorticoids each year. A step-wise approach is used to classify asthma severity and determine approach to treatment. Intermittent asthma is defined as having two or fewer daytime symptoms per week, two or fewer nighttime awakenings due to symptoms in the past month, use of short-term beta agonist rescue inhaler less than two times per week, no impairment in activities of daily living (ADLs) between exacerbations, home readings of lung function within the normal range (greater than or equal to 80% of normal), and zero or one exacerbation requiring oral glucocorticoids in the past year.

One Step Further Question: What is the most common inherited risk factor for thromboembolic disease?

Factor V Leiden

True or false: Bacterial infections are the most common cause of acute bronchitis?

False.

Rapid Review Foreign Body Aspiration

Foreign Body Aspiration Patient will be a child Complaining of a sudden episode of coughing, wheezing, or stridor Comments: most common is the right main bronchus

Question: What is the most common cause of pleural effusion in the U.S.?

Heart failure. Pleural Effusion Transudate: CHF (most common) Exudate: infection > malignancy, PE ↓ Breath sounds + dull percussion + ↓ tactile fremitus CXR: blunting of the costophrenic angle

Rapid Review Idiopathic Pulmonary Fibrosis

Idiopathic Pulmonary Fibrosis Patient will be a male With a history of smoking Complaining of chronic dry cough and dyspnea Labs will show restrictive characteristics - decrease FVC and FEV1 but a near normal FEV1/FVC ratio CXR will show "honeycombing" Treatment is O2, pulmonary rehabilitation

Question: What is the CURB-65 score?

It attempts to predict the severity of pneumonia to help determine inpatient vs. outpatient treatment. Legionella Pneumonia Patient will be complaining of fevers, malaise, myalgias, cough and GI symptoms Labs will show leukocytosis, elevated liver transaminases and hyponatremia CXR will show unilateral patchy alveolar lower lobe infiltrates Most commonly caused by gram-negative bacillus and is found in aquatic environments Treatment is azithromycin

One Step Further Question: What are the local and systemic side effects of inhaled corticosteroids?

Local: Cough, dysphonia, oropharyngeal candidiasis. Systemic: Adrenal suppression, osteoporosis, skin thinning, easy bruising and cataracts.

Question: The differential diagnosis of interstitial lung disease (ILD) includes which three major conditions?

Lung malignancy, lung infection and congestive heart failure. Idiopathic Pulmonary Fibrosis Patient will be a male With a history of smoking Complaining of chronic dry cough and dyspnea Labs will show restrictive characteristics - decrease FVC and FEV1 but a near normal FEV1/FVC ratio CXR will show "honeycombing" Treatment is O2, pulmonary rehabilitation

One Step Further Question: What are the contraindications to using antenatal corticosteroid therapy?

Maternal systemic infections, namely chorioamnionitis. Infant (Neonatal) Respiratory Distress Syndrome ↓ Fetal surfactant Risk factors: prematurity (most common), maternal diabetes, c-section Respiratory difficulty within a few hours after birth CXR: ground glass appearance Rx: O2, intubation, CPAP, surfactant Concern for premature birth: prenatal glucocorticoids O2 complications: blindness, bronchopulmonary dysplasia

Question: What is a side effect of dapsone that can lead to hypoxia?

Methemoglobinemia. Pneumocystis Pneumonia (PCP) Patient with a history of HIV Complaining of gradual onset of non-productive cough Labs will show CD4 < 200, increased LDH CXR will show bilateral infiltrates (bat wing pattern) Most commonly caused by Pneumocystis jirovecii Treatment is TMP-SMX

One Step Further Question: What are the three laboratory criteria for an exudative effusion?

Pleural:Serum Protein >0.5; Pleural:Serum LDH >0.6; Pleural fluid LDH greater than 2/3 upper limit of normal for serum LDH Pleural Effusion Transudate: CHF (most common) Exudate: infection > malignancy, PE ↓ Breath sounds + dull percussion + ↓ tactile fremitus CXR: blunting of the costophrenic angle

Question: What are the mechanisms of pulmonary bleeding?

Pulmonary hypertension, erosion into a blood vessel, and coagulopathy. Hemoptysis Diastolic murmur: mitral stenosis Sudden SOB, CP: PE Trauma: pulmonary contusion Immunodeficiency, immigrant: TB Hx of TB or sarcoidosis: aspergilloma Renal dysfunction: Goodpasture's syndrome or Wegener's granulomatosis (Granulomatosis with polyangiitis) Hx of tobacco use, weight loss: malignancy Massive hemoptysis: > 600 mL of blood/24 hrs Massive hemoptysis Rx: patient in bleeding side down position, mainstem bronchus intubation

Question: What diagnostic test, although low sensitivity, is often used when influenza is suspected?

Rapid influenza test.

One Step Further Question: What is the treatment of asbestosis?

Supportive care: oxygen, pulmonary rehabilitation, pneumococcal and influenza immunization and pulmonary function monitoring. Asbestosis Shipping, roofing, plumbing SOB + nonproductive cough + chronic hypoxia CXR: "ivory white" calcified pleural plaques Ferruginous bodies Bronchogenic carcinoma > mesothelioma From the roof, but affects the base (lower lobes)

Question: What is the treatment of choice for respiratory distress syndrome?

Surfactant. Transient Tachypnea of the Newborn Patient will be a child shortly after birth Complaining of sudden onset of difficulty breathing PE will show tachypnea, nasal flaring, retractions and hypoxia CXR will show parenchymal infiltrates and fluid in the pulmonary fissures Most commonly caused by residual pulmonary fluid Treatment is supportive, self-resolving within 72 hours Comments: Most common cause of neonatal respiratory distress

Question: How soon after blood transfusion initiation will anaphylaxis occur?

Within minutes. It is triggered by presence of IgA antibodies in IgA-deficient patients.

Question: What is the blood pressure goal for patients with diabetes mellitus type 2?

< 140/90.

Pneumoconiosis

Asbestosis: shipping, roofing, plumbing Berylliosis: aerospace, fluorescent bulbs Byssinosis: cotton Silicosis: foundries, sandblasting, mines Coal worker's lung: coal Siderosis: iron Stannosis: tin SOB + nonproductive cough + chronic hypoxia PFTs: reduced lung volumes CXR: interstitial fibrosis Asbestos: from the roof, but affects the base (lower lobes) Silica, coal: from the base (earth), but affect the roof (upper lobes) Cor pulmonale

Question: What is the antibiotic of choice for psittacosis in children and pregnant women?

Erythromycin. Psittacosis Patient with a history of exposure to birds Complaining of high fevers, severe headache, myalgias, nonproductive cough PE will show hepatosplenomegaly CXR will show patchy perihilar or lower lobe infiltrates Most commonly caused by Chlamydia psittaci Treatment is doxycycline

Question: What organism is associated with pneumonia in patients with COPD?

Haemophilus influenzae,Pseudomonas aeruginosa. Bacterial Pneumonia S. pneumonia: most common, rusty colored sputum, rigors, gram+ paired lancets Klebsiella: alcoholics, currant jelly sputum, bulging fissures, S. aureus: IVDA, postinfluenza, elderly, gram+ cocci in clusters H. influenzae: COPD, gram negative pleomorphic rods Pseudomonas: cystic fibrosis, nursing home resident and cyanosis Health care associated pneumonia: pseudomonas, MRSA Outpatient, healthy: macrolide or doxycycline Outpatient, comorbidity: respiratory tract fluoroquinolone (RTF) Inpatient: RTF ICU: antipneumococcal ß-lactam (ceftriaxone or cefotaxime) + either azithromycin or an RTF

Question: Why is airway compromise in epiglottitis less common in adults than in children?

In adults, there is an increased trachea diameter to epiglottis diameter making airway compromise unusual. Epiglottitis Patient will be complaining of rapid onset of fever and dysphagia PE will show patient leaning forward, drooling, inspiratory stridor Imaging will show "thumbprint" sign Most commonly caused by H. influenzae, Streptococcus Treatment is IV antibiotics and airway management

Question: What is the most common location of extrapulmonary TB?

Lymph nodes. Pulmonary Tuberculosis (TB) RFs: immunodeficiency, immigrant, close contact Latent/primary TB: asymptomatic Active/reactivation TB: fever, night sweats, weight loss, productive cough, hemoptysis Erythema nodosum Primary TB CXR: Ghon focus Active/reactivation TB: upper lobes, cavitary lesions Dx: sputum smears for acid-fast bacilli (AFB), sputum/tissue culture for AFB (gold standard) PPD: gold standard for latent TB dx Latent TB rx: 9 months of INH Primary TB rx: rifampin, INH, pyraziniamide, ethambutol (RIPE)

Question: What are the most common sites of hemorrhage in massive hemoptysis?

Massive hemoptysis typically involves the bronchial or pulmonary arteries.

What is the common side effect of rifampin?

Orange discoloration of body fluids. Pulmonary Tuberculosis (TB) RFs: immunodeficiency, immigrant, close contact Latent/primary TB: asymptomatic Active/reactivation TB: fever, night sweats, weight loss, productive cough, hemoptysis Erythema nodosum Primary TB CXR: Ghon focus Active/reactivation TB: upper lobes, cavitary lesions Dx: sputum smears for acid-fast bacilli (AFB), sputum/tissue culture for AFB (gold standard) PPD: gold standard for latent TB dx Latent rx: 6-9 months of INH Primary rx: Rifampin, INH, Pyrazinimide, Ethambutol (RIPE)

Question: Name 2 clinical prediction rules that may be used to help determine whether a patient with CAP requires inpatient or outpatient management?

PORT/PSI (Pneumonia Severity Index) and the CURB 65.

Pneumocystis Pneumonia (PCP)

Pneumocystis Pneumonia (PCP) Patient with a history of HIV Complaining of gradual onset of non-productive cough Labs will show CD4 < 200, increased LDH CXR will show bilateral infiltrates (bat wing pattern) Most commonly caused by Pneumocystis jirovecii Treatment is TMP-SMX

Where are most aspirated foreign bodies located?

Right main bronchus. Foreign Body Aspiration Patient will be a child Complaining of a sudden episode of coughing, wheezing, or stridor Comments: most common is the right main bronchus

Question: In a patient with AIDS, what medication is given prophylactically at a CD4 count of less than 100 cells/μL to prevent infection with toxoplasmosis gondii?

Trimethoprim/sulfamethoxazole

Question: True or false: All patients with chronic obstructive pulmonary disease should be prescribed a short-acting bronchodilator?

True.

Question: What is the choroidal tubercle?

A granuloma in the choroid of the retina that is specific for disseminated TB. Pulmonary Tuberculosis (TB) RFs: immunodeficiency, immigrant, close contact Latent/primary TB: asymptomatic Active/reactivation TB: fever, night sweats, weight loss, productive cough, hemoptysis Erythema nodosum Primary TB CXR: Ghon focus Active/reactivation TB: upper lobes, cavitary lesions Dx: sputum smears for acid-fast bacilli (AFB), sputum/tissue culture for AFB (gold standard) Latent TB dx: PPD or interferon-gamma release assay Latent rx: 9 months of INH Primary rx: Rifampin, INH, Pyrazinamide, Ethambutol (RIPE)

Question: What arterial blood gas finding is concerning for impending respiratory failure in an asthmatic?

A normal, elevated or rising carbon dioxide. Asthma Asthma: airway inflammation + bronchial hyperresponsiveness + reversible airflow obstruction PEF <50%: severe exacerbation Treatments: O2: maintain SpO2 > 88% ß-agonists: ↑ cAMP → bronchodilation Anticholinergics: ↓ bronchoconstriction Corticosteroids: ↓ inflammation, administer early Mg: severe exacerbations Non-invasive ventilation: ↓ work of breathing Mechanical ventilation: Objective: maximize expiratory time Low respiratory rate High inspiratory flow rate Maintain plateau pressure <30 cm H2O Permissive hypercapnia to avoid breath stacking

A 65-year-old smoker with no documented cardiac disease presents with several months of worsening cough and exertional dyspnea. He denies exposure to inhalation toxins or dusts. Examination reveals bibasilar inspiratory crackles and fingernail clubbing. Spirometry reveals a restrictive lung pattern. A chest computed tomography scan is read as "honeycombing in the periphery and bases." A bronchoalveolar lavage report is only significant for lymphocytosis and a histological classification of usual interstitial pneumonia. Which of the following is the most likely diagnosis? Chronic obstructive pulmonary disease Community-acquired pneumonia Idiopathic pulmonary fibrosis Pneumoconiosis

Correct Answer ( C ) Explanation: Nomenclature of restrictive pulmonary disease can be confusing. It can be simplified if one first dissects the lung into airspace (mostly obstructive diseases), parenchyma (the lung "meat" which doesn't contain air, aka interstitium) and chest wall (the mechanism that runs the airspaces and parenchyma). Restrictive disease encompasses abnormalities of the parenchyma and chest wall. Idiopathic pulmonary fibrosis (IPF) represents a main subclassification of the more than 200 interstitial lung diseases. IPF is a chronic progressive, and often fatal, interstitial lung disease which affects adults over 50 years of age (men > women and smokers > non-smokers). Although linked to tobacco, occupational toxin exposure, gastroesophageal reflux and genetic predisposition, the exact etiology remains unknown, hence the term idiopathic. It is felt the pathology is based on abnormal wound healing, oxidant-antioxidant imbalance and excessive deposition of collagen. Patients experience a progressive dry cough, exertional dyspnea, inspiratory "velcro-like" crackles and fingernail clubbing. Radiography is nonspecific, but high resolution computed tomography reveals the diagnostic findings of fibrotic changes referred to as "honeycombing" (small cystic spaces enveloped by thick, fibrotic, well-defined walls, mainly occurring in the periphery and bases). Bronchoalveolar lavage (BAL) is an important diagnostic test, however, it is mainly used to rule-out other pulmonary disease such as malignancy and infections. In IPF, the tissue obtained from lung washings will show a histological appearance of usual interstitial pneumonia, the classic pathological description of IPF.

One Step Further Question: What is the most common form of lung cancer?

Adenocarcinoma.

Question: What is aspiration pneumonia?

An alveolar space infection resulting from the inhalation of pathogenic material from the oropharynx. Aspiration Pneumonitis Inflammatory chemical injury Inhalation of gastric contents Can lead to aspiration pneumonia Depressed level of consciousness Unilateral focal/patchy consolidations in dependent lung segments Prophylactic antibiotics NOT recommended

Question: What is a saddle pulmonary embolism?

An embolus that lodges in the bifurcation formed by the main pulmonary artery and right and left pulmonary arteries. Pulmonary Embolism 95% arise from deep leg veins Sudden onset of symptoms in 50% SOB, CP, tachypnea ECG: sinus tachycardia, nonspecific ST-T changes, right heart strain, S1Q3T3 (classic finding) CXR: nonspecific abnormalities, Hampton's hump (pleural-based wedge infarct), Westermark's sign (vascular cut-off sign) V/Q scan: usually nondiagnostic Low clinical suspicion: negative D-dimer excludes PE Dx of choice: CTPA Treatment: Anticoagulation Thrombolytics (if massive and HD unstable or submassive with shock, respiratory failure or evidence of moderate to severe RV strain) Embolectomy (last resort)

One Step Further Question: True or false: Pneumocystis jiroveci is classified as a fungus based on ribosomal DNA and cell wall composition

Answer: True.

One Step Further Question: What is a Westermark sign on X-ray?

Answer: Westermark sign is a rare indication of pulmonary embolus (PE) on X-ray. The X-ray would show a sharp cut-off of pulmonary vessels with distal hypoperfusion.

One Step Further Question: What is the difference between aspiration pneumonitis and aspiration pneumonia

Aspiration pneumonitis is an inflammatory reaction which results from the aspiration of gastric acidic fluid from the stomach. Aspiration pneumonia is a bacterial pneumonia caused by aspiration. Bacterial Pneumonia S. pneumonia: most common, rusty colored sputum, rigors, gram+ paired lancets Klebsiella: alcoholics, currant jelly sputum, bulging fissures, S. aureus: IVDA, postinfluenza, elderly, gram+ cocci in clusters H. influenzae: COPD, gram negative pleomorphic rods Pseudomonas: cystic fibrosis, nursing home resident and cyanosis Health care associated pneumonia: pseudomonas, MRSA Outpatient, healthy: macrolide or doxycycline Outpatient, comorbidity: respiratory tract fluoroquinolone (RTF) Inpatient: RTF ICU: antipneumococcal ß-lactam (ceftriaxone or cefotaxime) + either azithromycin or an RTF

Which of the following is the most common dysrhythmia associated with the diagnosis of pulmonary embolism? Atrial fibrillation AV-nodal reentrant tachycardia Multifocal atrial tachycardia Sinus tachycardia

Atrial fibrillation (A) can be seen with pulmonary embolism but is less common than sinus tachycardia. Atrioventricular nodal reentry tachycardia (AVNRT) (B) is the most common type of reentrant supraventricular tachycardia. Pulmonary embolism is an unlikely cause of AVNRT Multifocal atrial tachycardia (MAT) (C) is a dysrhythmia that is seen in a variety of clinical disorders. It is more common in the elderly and is associated with COPD and hypoxia. In addition to a heart rate greater than 100 beats per minute, the characteristic ECG feature is variability in P wave morphology, with each unique P wave morphology felt to indicate a different site of atrial origin. Patients with multiple P wave morphologies but a normal heart rate are considered to have a wandering atrial pacemaker because the heart rate does not meet criteria for a tachycardia.

Question: Name two steroid-sparing alternative medications used for pulmonary sarcoidosis.

Azathioprine and methotrexate. Sarcoidosis African-Americans, females Primary target organ: lungs Parotid enlargement Hypercalcemia CXR: bilateral hilar adenopathy Biopsy: noncaseating granulomas Steroids

Bacterial Pneumonia Overview

Bacterial Pneumonia S. pneumonia: most common, rusty colored sputum, rigors, gram+ paired lancets Klebsiella: alcoholics, currant jelly sputum, bulging fissures, S. aureus: IVDA, postinfluenza, elderly, gram+ cocci in clusters H. influenzae: COPD, gram negative pleomorphic rods Pseudomonas: cystic fibrosis, nursing home resident and cyanosis Health care associated pneumonia: pseudomonas, MRSA Outpatient, healthy: macrolide or doxycycline Outpatient, comorbidity: respiratory tract fluoroquinolone (RTF) Inpatient: RTF ICU: antipneumococcal ß-lactam (ceftriaxone or cefotaxime) + either azithromycin or an RTF

Question: What is a physiologic complication of noninvasive positive pressure ventilation?

Barotrauma including pneumothorax.

Question: What is the prevalence of idiopathic pulmonary fibrosis in the U.S.A?

Between 14 and 43 per 100,000 persons in the US. Idiopathic Pulmonary Fibrosis Patient will be a male With a history of smoking Complaining of chronic dry cough and dyspnea Labs will show restrictive characteristics - decrease FVC and FEV1 but a near normal FEV1/FVC ratio CXR will show "honeycombing" Treatment is O2, pulmonary rehabilitation

Question: What pulmonary finding is pathognomonic for sarcoidosis?

Bilateral hilar adenopathy.

Question: What is the most useful test to confirm a diagnosis of sarcoidosis?

Biopsy of the affected organ. Sarcoidosis African-Americans, females Primary target organ: lungs Parotid enlargement Hypercalcemia CXR: bilateral hilar adenopathy Biopsy: noncaseating granulomas Steroids

A 35-year-old woman comes to the clinic complaining of difficulty seeing, blurred vision, eye pain, and cough. She describes the cough as being dry and nonproductive. She has no past medical history and takes no medications. Ophthalmologic examination shows uveitis. Chest X-ray reveals bilateral hilar adenopathy. Which of the following laboratory findings would also most likely be associated with this patient's condition? Elevated C-reactive protein Elevated ESR Elevated serum angiotensin converting enzyme levels Increased sweat chlorid

Correct Answer ( C ) Explanation: Uveitis, bilateral hilar adenopathy, and a dry cough most likely indicate sarcoidosis. Sarcoidosis is granulomatous disease that can form nodules in multiple organs. Patients with sarcoidosis typically present with fatigue, weight loss, arthritis, dry eyes, blurry vision, and respiratory symptoms (eg, cough, dyspnea). Management usually involves the use of corticosteroids (eg, prednisone). Elevated levels of serum angiotensin converting enzyme (ACE) are typically elevated in about 75% of patients with sarcoidosis. However, elevated levels of ACE can also be seen in other conditions, such as, asbestosis, diabetes, and lung cancer. Therefore, it is not routinely used as a diagnostic test.

A 55-year-old obese man presents to the clinic complaining of fatigue and poor concentration for the past six months. His wife reports that he snores nightly and often sounds like he gasping for air. Which of the following would most likely be found on physical exam? A large neck circumference Coarsening of facial features Increased skin pigmentation Moon facies

Correct Answer ( A ) Explanation: A large neck circumference would most likely be found on physical exam in this man with symptoms concerning for obstructive sleep apnea. Cardinal features of obstructive sleep apnea include irregular and abnormal respiratory patterns, loud snoring and snorts during sleep. Symptoms attributable to disrupted sleep include sleepiness, fatigue and poor concentration. Well-defined risk factors for obstructive sleep apnea include older age, male gender, obesity, craniofacial abnormalities, and upper airway soft tissue abnormalities. Tonsillar hypertrophy, macroglossia, retrognathia/micrognathia, and upper airway mass lesions can contribute to upper airway narrowing. The physical exam can be normal, although obesity, elevated blood pressure, a narrow airway, and a large neck circumference are common. Obstructive sleep apnea is more strongly correlated with an increased neck size or waist circumference than general obesity and is particularly prominent among men who have a collar size greater than 17 inches and women who have a collar size greater than 16 inches. Moon facies (D) is associated with Cushing syndrome, not obstructive sleep apnea. Increased skin pigmentation (C) is sometimes seen in Addison disease and coarsening of facial features (B) is seen with acromegaly. Neither of these physical exam findings are associated with obstructive sleep apnea.

A 16-year-old boy with a history of asthma presents complaining of increasing episodes of evening and daytime symptoms. He is on a short acting inhaled beta agonist on an as needed basis. He presently needs to use his short acting beta agonist daily. Which of the following is the most appropriate addition to this patient's medication regimen? Inhaled corticosteroid Leukotriene inhibitor Long acting beta agonist inhaler Methylxanthine oxidase inhibitor

Correct Answer ( A ) Explanation: According to the stepwise approach for managing asthma by the National Asthma Education and Prevention Program, inhaled corticosteroids are indicated for mild to moderate persistent asthma. For most patients twice-daily dosing provides adequate control of asthma symptoms. Short acting beta agonists are helpful with controlling acute exacerbations by acting as bronchodilators; however they are not helpful in the long term management in patients with persistent asthma. Using an inhalation chamber, also known as a "spacer," along with rinsing of the month after inhaled corticosteroid use decreases local side effects and systemic absorption.

A 43-year old man presents complaining of a two-week history of gradually worsening dry cough, fatigue and occasional shortness of breath. He has "felt warm" but has not checked his temperature. Review of systems is notable for mild diarrhea and decreased appetite, though he is drinking fluids well. He denies chronic medical problems and takes no medications. He does not smoke. His temperature is 100.6°F; pulse 112; BP 122/78; RR 24; pulse oximetry is 92% on room air. He appears tired, though is not ill-appearing. Lung-fields sound clear on auscultation, though a chest X-ray is obtained which demonstrates diffuse infiltrates. What is the most appropriate treatment for his condition? Azithromycin Metronidazole Olsetamivir Piperacillin and Tazobactam

Correct Answer ( A ) Explanation: Azithromycin, a macrolide antibiotic, is the most appropriate choice for this patient with community-acquired bacterial pneumonia (CAP). The most common side effects of azithromycin include nausea, vomiting and diarrhea, though other side effects such as headache and fatigue may occur. Other reasonable choices for treatment would include doxycycline or a fluoroquinolone. The most common organism responsible for "atypical" bacterial pneumonia is Mycoplasma pneumoniae, though suspicion for a particular pathogen should be guided by the patient's co-morbidities as well as setting. In contrast to CAP caused by typical pathogens such as S. pneumoniae or H.influenzae, this type of bacterial pneumonia usually presents more gradually with symptoms of dry cough, dyspnea, and extra-pulmonary symptoms such as headache, myalgias, fatigue, and GI disturbance. Other bacterial organisms causing pneumonia in certain hosts and settings may include Legionella, Moraxella, Staphylococcus, Chlamydia, Pseudomonas, Klebsiella, Acinetobacteria, and Mycobacterium sp.

A 53-year-old man comes to the emergency department complaining of a 5-day history of a cough and shortness of breath. His temperature is 37.6°C (99.8°F). Auscultation of the lungs shows rhonchi and wheezing. Chest X-ray shows thickening of the bronchial walls in both lower lobes. Laboratory studies show a slightly elevated white blood count. Which of the following is the most likely diagnosis? Acute bronchitis Asthma Influenza Pneumonia

Correct Answer ( A ) Explanation: Based on the constellation of findings, this patient most likely has acute bronchitis. Acute bronchitis is a self-limited inflammation of the bronchi and typically presents with mucopurulent cough for more than 5 days. Chest X-ray will show thickening of the bronchial walls in the lower lobes and auscultation of the lungs usually reveals wheezing and rhonchi. Acute bronchitis is typically caused by viruses (eg, respiratory syncytial virus, rhinovirus, influenza), therefore, antibiotics are not indicated. Most cases of acute bronchitis are self-limited and resolve on their own within a few weeks. Although influenza (C) can cause acute bronchitis, it is not likely this patient has influenza. Influenza typically presents with cough, fever, sputum production, and constitutional symptoms. Pneumonia (D) presents similarly to influenza. Radiography typically shows lung consolidation (ie, lobar pneumonia). It is unlikely this patient has influenza or pneumonia due to the lack of fever and other constitutional symptoms. New onset asthma (B) is not likely in this patient because it is less frequent in older adults and usually presents before the age of 7.

A patient presents with acute dyspnea and pleuritic chest pain. You suspect pulmonary embolism. Which of the following is the most appropriate test to confirm the diagnosis of a pulmonary embolus? Computed tomography angiography D-dimer assay Echocardiography Ventilation-perfusion scan

Correct Answer ( A ) Explanation: Clinical impression alone does not suffice for the diagnosis of acute pulmonary embolism. Diagnosis is difficult due to the low specificity of the common presenting symptoms of acute dyspnea, cough and pleuritic chest pain. Therefore, supplemental testing is used to effectively rule-in or rule-out pulmonary embolism (PE). Routine laboratory tests are nonspecific. Arterial blood gas measurements have a limited role in diagnosing PE. Electrocardiogram and chest radiography abnormalities found in patients with PE are also commonly found in patients without PE, which limits the diagnostic usefulness of these tests. Pulmonary angiography is the definitive, or "gold standard", test for the diagnosis of PE. It is performed by injecting contrast intravenously via the femoral vein. The radiologist looks for a filling defect or abrupt cutoff in the pulmonary vasculature which concretely indicates the presence of an embolus. Helical CT with pulmonary angiography is being used more and more as the diagnostic test for PE as it can detect not only an embolus, but also alternative pulmonary abnormalities which may be causative of the patient's symptoms. However, one must take into account the radiation risk of CT versus the diagnostic benefit or necessity of this test.

Which of the following is the most common cause of cor pulmonale? Chronic obstructive pulmonary disease Connective tissue disorders Left-sided cardiac disease Right ventricular hypertrophy

Correct Answer ( A ) Explanation: Cor pulmonale is a condition of altered right heart function due to respiratory disease, and is estimated to account for 7% of all types of adult heart disease in the U.S. Acute forms are due to pulmonary emboli and acute respiratory distress syndrome. On the other hand, cor pulmonale more commonly has a chronic progressive course. Some pathophysiologic mechanisms include pulmonary vasoconstriction, pulmonary vascular bed compromise due to pulmonary interstitial or alveolar disease, connective tissue disease and high blood viscosity. There are several underlying etiologies, the most common being COPD (>50%). Others include pulmonary hypertension, rheumatoid disease, pulmonary thromboemboli, polycythemia vera, and sickle cell disease. Cor Pulmonale Pulmonary HTN + RVH → right heart failure MC chronic cause: COPD MC acute cause: PE Right heart catheterization

Which of the following HIV-positive patients suspected of having Pneumocystis pneumonia (PCP) should receive prednisone before treatment with trimethoprim/sulfamethoxazole? A 10-year-old with a normal chest x-ray and a PaO2 of 65 mm Hg A 15-year-old with diffuse interstitial infiltrates on chest x-ray, a pulse oximetry of 92% and PaO2 of 80 mm Hg A 20-year-old with diffuse interstitial infiltrates on chest x-ray and an A-a gradient of 25 mm Hg An 18-year-old with diffuse interstitial infiltrates on chest x-ray, a pulse oximetry of 94% on room air, PaO2 of 75 mm Hg, and an A-a gradient of 10 mm Hg

Correct Answer ( A ) Explanation: Corticosteroids are used as adjunct therapy in HIV-positive patients with moderate to severe PCP (now known as Pneumocystis jiroveci pneumonia), defined by a room air arterial oxygen partial pressure (PaO2) of less than 70 mm Hg (severe disease is less than 60 mm Hg) or an alveolar-arterial oxygen gradient that exceeds 35 mm Hg (exceeding 45 mm Hg for severe disease). When administered, steroids should be given before trimethoprim/sulfamethoxazole or pentamidine because microbial degradation and clearance caused by antibiotics may trigger a severe inflammatory response. Corticosteroid therapy can blunt this inflammatory response, improve oxygenation, and reduce the incidence of respiratory failure.

17-year-old girl with a history of well-managed cystic fibrosis is being evaluated for a steadily worsening chronic cough with shortness of breath and wheezing. She is producing copious purulent malodorous sputum and occasional hemoptysis. Crackles are heard at her bilateral lung bases. Which of the following findings would be most expected on this patient's chest radiograph? Dilated, thickened bronchi with "tram-track" marks Low lung volumes and ground glass opacities Lung hyperinflation with flattening of the diaphragm Normal chest radiograph

Correct Answer ( A ) Explanation: Dilated, thickened bronchi are classic radiograph findings in patients with bronchiectasis, the most likely disease in this patient. The bronchi are often described as having "tram-track" or ring-like markings. Scattered, irregular opacities, atelectasis, or focal consolidations may also be noted. A suspicion of bronchiectasis based on radiographic findings and patient symptoms warrants a high-resolution CT scan which is a more precise diagnostic study. Bronchiectasis is suspected in patients who have a chronic cough, dyspnea, wheezing, and heavy production of purulent, foul-smelling sputum. Pleuritic chest pain, weight loss, and anemia are commonly associated. An exam usually reveals crackles at the lung bases and nail clubbing in severe disease. In the United States, the greatest percentage of patients with bronchiectasis develop it secondary to cystic fibrosis. Lung infections, tumor presence, and immunodeficiency states are other less common causes. Antibiotics are usually needed with the choice being guided by sputum cultures. Haemophilus influenza, Streptococcus pneumonia, and Staphylococcus aureus are often isolated. In addition to proper antibiotic coverage, bronchiectasis should be treated with daily chest physiotherapy and inhaled bronchodilators.

A 45-year-old woman presents to the Emergency Department with a fever, productive cough, and diarrhea. Her chest X-ray shows a patchy unilobar infiltrate and her sodium is 127 mmol/L. Which of the following organisms is most likely to be responsible for these symptoms? Legionella pneumophila Pseudomonas aeruginosa Staphylococcus aureus Streptococcus pneumoniae

Correct Answer ( A ) Explanation: Legionella pneumophila is the most likely organism and causes Legionnaires' disease. It is responsible for about 5% of all pneumonias. Common symptoms include a fever, cough, diarrhea, and confusion. Chest X-ray findings vary but often show a patchy unilobar infiltrate. Laboratory abnormalities are common with hyponatremia being the classic finding. First-line treatment includes either respiratory quinolones (e.g. levofloxacin) or macrolides (e.g. azithromycin). Streptococcus pneumoniae (D) is the most common cause of community-acquired pneumonia (CAP). It presents classically with fever, chills, productive cough, and pain and a lobar opacity on chest X-ray. First-line treatment for inpatient management is a third-generation cephalosporin (e.g. ceftriaxone or cefotaxime). Outpatient pneumococcal pneumonia can be treated with amoxicillin or a macrolide. Pseudomonas aeruginosa (B) is a common cause of hospital-acquired pneumonia. It should be considered in patients with cystic fibrosis, bronchiectasis, or an immunocompromised state. Treatment can include an antipseudomonal penicillin (e.g. piperacillin-tazobactam) or an antipseudomonal cephalosporins (e.g. cefepime). Staphylococcus aureus (C) is a common cause of healthcare-acquired pneumonia and is seen following influenza infections. Treatment includes either vancomycin or linezolid since methicillin resistance is common.

17-year-old girl with a history of asthma presents to your office with complaints of wheezing and shortness of breath. She says that in the past month she has experienced symptoms approximately 3-4 times per week requiring use of her short-acting beta agonist inhaler. She has also woken up at night four times during the month with shortness of breath and occasionally gets dyspneic on her daily walk. Which of the following is the most effective management? Add a low dose inhaled glucocorticoid Add a low dose inhaled glucocorticoid plus long-acting beta agonist Continue use of short-acting beta agonist only Start an oral course of glucocorticoids

Correct Answer ( A ) Explanation: Mild persistent asthma is defined as having symptoms more than twice per week but less than daily, 3-4 nocturnal awakenings per month, symptoms requiring the use of a short-acting beta agonist rescue inhaler more than two times per week, minor interference with activities of daily living, FEV1 measurements and FEV1/FVC ratio within normal limits and two or more asthma exacerbations requiring an oral course of corticosteroids per year. Treatment of asthma follows a step-wise plan depending on the severity of symptoms. Mild persistent asthma requires daily use of a long-term controlling medication and a low dose inhaled glucocorticoid. Use of a short-acting beta agonist rescue inhaler should also continue as needed.

65-year-old obese man presents to your office with complaints of snoring and fatigue. His wife makes him sleep in a separate room because of his loud snoring and gasping for air in his sleep. Which of the following is the most appropriate therapy? Continuous positive airway pressure therapy Modafinil Oxygen therapy Theophylline

Correct Answer ( A ) Explanation: Obstructive sleep apnea (OSA) is a disorder characterized by a decrease or cessation of airflow during sleep. It is more common in men. The prevalence increases with age and obesity is the most well-documented risk factor. The classic symptoms of OSA are snoring, spousal report of apnea, daytime sleepiness, fatigue and hypoxia. Diagnosis of OSA is confirmed by a sleep study. All patients with OSA should be counseled on behavioral modifications, including weight loss, sleeping in the lateral recumbent position, smoking cessation, and avoiding alcohol for 4-6 hours prior to bedtime. The cornerstone of OSA treatment is continuous positive airway pressure (CPAP) therapy. Over time, untreated OSA may lead to increased morbidity and mortality due to hypertension, heart disease, stroke, diabetes or metabolic syndrome.

33-year-old man presents to the ED with several weeks of cough, pleuritic chest pain, weight loss, and night sweats. The patient drinks a 6-pack of beer daily. Vital signs are BP 145/75, HR 88, RR 18, and T 37.7°C. Pulmonary exam reveals crackles and decreased breath sounds on auscultation. You obtain the radiograph seen above. Which of the following is the most likely diagnosis? ALung abscess BNeoplasm CSeptic pulmonary emboli DTuberculosis

Correct Answer ( A ) Explanation: Patients with lung abscess classically present with several weeks of cough, fever, pleuritic chest pain, weight loss, and night sweats. There may be cough productive of putrid sputum. Because the progression of lung abscess is indolent, tachycardia, tachypnea, or fever may be absent. The chest radiograph often shows an area of dense consolidation with an air-fluid level inside a thick-walled cavitary lesion. Those who abuse alcohol or have other conditions associated with the potential for aspiration are at greatest risk for lung abscess development.

A 72-year-old woman was hospitalized for atrial fibrillation with rapid ventricular rate. She is being discharged today on amiodarone for long term control of her dysrhythmia. Which of the following diagnostic studies should be followed as an outpatient? Chest X-ray Coagulation studies Complete blood counts Renal panels

Correct Answer ( A ) Explanation: Pulmonary function tests should be monitored for patients on chronic amiodarone therapy. Amiodarone is a class III anti-arrhythmic drug used to treat many common dysrhythmias. Annual chest radiograph is are recommended when patients are on chronic Amiodarone use. Pulmonary toxicity generally correlates more closely with the total cumulative dose than the serum levels. It usually occurs months to years after initiation. Several types of pulmonary toxicity may result from chronic amiodarone therapy; however, the most common is a chronic interstitial pneumonitis. A non-productive cough and dyspnea are present in the majority of affected individuals. If signs or symptoms develop the patient should be referred for pulmonary function testing and amiodarone should be immediately discontinued. Other complications of chronic amiodarone therapy include, but are not limited to, thyroid dysfunction, both hypo- and hyperthyroidism, symptomatic hepatitis, corneal microdeposits, optic neuropathy, or dermatologic manifestation such as photosensitivity and blue-gray skin discoloration. Thyroid function tests and hepatic panels are recommended every six months. Dermatologic physical exams and ophthalmologic eye evaluations are recommended as needed for signs or symptoms. These complications are treated by either reducing the dose and discontinuation.

What lab abnormality is commonly seen in sarcoidosis? Hypercalcemia Hypocalcemia Hypophosphatemia Neutropenia

Correct Answer ( A ) Explanation: Sarcoidosis has a number of lab abnormalities associated with it including hypercalcemia. Sarcoidosis is a multisystem inflammatory disease that involves non-caeseating granulomas composed of collections of T-helper cells and other inflammatory cells. It predominantly affects the lungs and intrathoracic lymph nodes but can involve a number of systems including cardiac, skin and neurologic. Chest radiography often reveals hilar adenopathy in early stages and progresses to pulmonary fibrosis. Patients may present with a host of non-specific symptoms including fever, fatigue, weight loss and polyarthritis. Other symptoms depend on end organ involvement. The granulomas secrete 1,25 vitamin D leading to hypercalcemia and hypercalciuria.

A 16-year-old man presents to the ED complaining of 3 days of nasal rhinorrhea, cough, myalgias, and generalized malaise. After a coughing episode yesterday, he developed pleuritic chest pain radiating to the left neck. His vital signs are BP 130/70 mm Hg, HR 76, RR 16, T 36.6°C and pulse oximetry 98% on room air. You obtain the chest radiograph seen above. Which of the following is the most appropriate next step in management? Discharge home with close follow-up Endoscopy Needle decompression Tube thoracostomy

Correct Answer ( A ) Explanation: Spontaneous pneumomediastinum is the presence of air or gas in the mediastinum. The source of air usually originates from the respiratory or gastrointestinal tract. Spontaneous pneumomediastinum usually results from rupture of alveoli, particularly after a strenuous Valsalva maneuver, leading to air dissecting along peribronchovascular sheaths and spreading into the mediastinum. The air then moves through mediastinal fascia planes and spreads to subcutaneous tissues of the thorax, upper limbs, and cervical region. Patients commonly report transient stabbing chest pain that may radiate to the shoulders, arms, or back. The finding of Hamman's sign (crunching sound heard on auscultation of the mediastinum with each heartbeat) is suggestive of pneumomediastinum but has a low sensitivity. Spontaneous pneumomediastinum is generally a benign, self-limited condition that usually does not require hospital admission. Secondary causes of pneumomediastinum such as Boerhaave's syndrome are more serious, and treatment is aimed at the underlying disorder.

13-year-old boy presents to your ED with worsening cough and exercise intolerance. The triage note says he has seen his primary care physician twice over the last month and was given albuterol which has not improved his symptoms. His vital signs show a temperature of 37.2°C, heart rate of 110 beats per minute, respiratory rate of 28 breaths per minute, blood pressure of 110/82 mm Hg, and pulse oximetry of 93% on room air. A chest X-ray obtained from triage is shown above. Which of the following findings do you expect on your physical examination? Coarse crackles Pleural friction rub Rhonchi Stridor

Correct Answer ( A ) Explanation: This patient has evidence of pulmonary venous congestion and cardiomegaly on X-ray, consistent with congestive heart failure (CHF) due to dilated cardiomyopathy. Dilated cardiomyopathy is the most common form of cardiomyopathy in children and the cause is unknown in about half of the cases. It causes dilation of all four cardiac chambers and significantly weakens systolic contraction leading to symptoms of congestive heart failure. These patients can present with both symptoms of left sided heart failure such as pulmonary coarse crackles, venous congestion, fatigue, or tachypnea and dyspnea on exertion, and right-sided congestive heart failure with hepatomegaly and peripheral edema. While peripheral edema may present as jugular venous distention and ankle edema in adults, this is less common in younger children who usually present with periorbital edema. Patients with new pulmonary venous congestion require a thorough cardiac work-up and admission to an intensive care unit. On auscultation, pulmonary edema is associated with coarse crackles, usually discontinuous and start early in inspiration and extend into expiration. Stridor (D) describes a high-pitched, monophonic sound made when breathing that is best heard over the anterior neck. It is associated with many conditions such as epiglottitis, croup, tracheitis, and foreign body obstruction.

Which of the following characteristics of transfusion-related acute lung injury (TRALI) differentiates it from transfusion-associated circulatory overload (TACO)? High fever Hypertension Hypoxemia Pulmonary edema

Correct Answer ( A ) Explanation: Transfusion-related acute lung injury (TRALI) complicates transfusions in 1 of every 5,000 units of PRBCs. Although rare, it is associated with high mortality. The pathophysiology is thought to be due to an acute neutrophilic response that leads to endothelial damage and massive capillary leak in the pulmonary vasculature. The diagnosis is clinical with patients presenting with signs and symptoms of acute lung injury or acute respiratory distress syndrome. TRALI is characterized by hypoxemia (oxygen saturation <90% on room air), and bilateral pulmonary infiltrates presenting within 6 hours of blood transfusion. There should not be any evidence of pulmonary artery hypertension or generalized fluid overload. Hypotension, tachycardia, and fever are common. Fever is not common in transfusion-associated circulatory overload (TACO). Treatment of TRALI is supportive, with low tidal volumes for mechanical ventilation and maintenance of euvolemia. Despite the initial appearance of fluid overload, these patients are normovolemic or intravascularly depleted secondary to the increase in vascular permeability and extravasation. Diuresis may lead to further deterioration and shock.

A 26-year-old gravida 3 para 2 at 12 weeks gestation presents with fever, myalgias, headache, and malaise. There have been multiple cases of influenza in the community and her influenza swab is positive. Which one of the following is recommended by the Centers for Disease Control and Prevention in this situation? Acyclovir Oseltamivir Rimantadine Supportive therapy only

Correct Answer ( B ) The Center for Disease Control recommends that neuraminidase inhibitors be prescribed for pregnant women and for those up to two weeks postpartum who have suspected or confirmed influenza. Women can continue to breastfeed while being treated with antivirals. Treatment with antiviral agents, should be started within 48 hours of symptom onset. The recommended treatment of influenza in pregnancy is one of the neuraminidase inhibitors, such as oseltamivir. Neuraminidase inhibitors have modest effectiveness in reducing influenza-related symptoms in patients and they protect against both strains of influenza A and influenza B.

When diagnosing pneumonia in children, which of the following findings has the highest odds ratio? Crackles Oxygen saturation < 92% Retractions Temperature >38°C

Correct Answer ( B ) Explanation: A study of more than 2,500 pediatric patients who presented to an ED showed hypoxia with oxygen saturation at or below 92% to have an odds ratio (OR) of 3.7 for pneumonia. This was higher than the OR for any other measured parameter. Pediatric Pneumonia 0-3 weeks: GBS 3 weeks-3 months: C. trachomatis 1 month-5 years: RSV 6-8 years: M. pneumoniae C. trachomatis: staccato cough, eye findings S. pneumoniae: rust-colored sputum M. pneumoniae: bullous myringitis ((classically associated together however middle ear aspirate culture results show that typical acute otitis media pathogens are the true cause)

A 22-year-old man presents with an asthma exacerbation. Which of the following is true regarding treatment modalities? Anticholinergic agents promote bronchodilation by decreasing cyclic adenosine monophosphate Beta-2-agonists promote bronchodilation by increasing cyclic adenosine monophosphate The onset of action for beta-2-agonists is 10 minutes The peak effect of corticosteroids occurs within 3-5 hours

Correct Answer ( B ) Explanation: Asthma is responsible for nearly two million Emergency Department visits per year. The mainstay of treatment consists of inhaled beta-2-agonists, inhaled anticholinergics, and corticosteroids. The combination of these drugs results in decreased rates of hospitalization and improvements in pulmonary function in the first 90 minutes. Beta-2-agonists promote bronchodilation by increasing cyclic adenosine monophosphate (cAMP). Their primary effect is on small peripheral airways with onset of action in less than five minutes. Studies have shown that administration of beta-2-agonists via metered-dose inhaler with a spacer provides similar bronchodilation when compared with nebulization. While use of the MDI may be more economical and take less time, it does require more supervision to ensure adequate use. Anticholinergic agents promote bronchodilation by decreasing cyclic guanosine monophosphate (cGMP), not cyclic adenosine monophosphate (A). Their primary effect is on the larger airways. Onset of action is also longer than with beta-2-agonists, taking up to one to two hours for peak effect. Corticosteroids work by inhibiting recruitment of inflammatory cells. They are indicated in all cases of moderate to severe asthma and in those who demonstrated limited response to beta-2-agonists. Onset of action is one to two hours with peak effect in about 24 hours, not three to five hours (C). Many studies have shown that oral and intravenous steroids provide equal benefit. Onset of action for beta-2-agonists is less than five minutes, not 10 minutes (D).

56-year-old man comes to the clinic complaining of a 6-day history of a mucopurulent cough and shortness of breath. His temperature is 37.6°C (99.8°F). Auscultation of the lungs reveals rhonchi and wheezing. Chest X-ray shows thickening of the bronchial walls in both lower lobes. Laboratory studies show a slightly elevated white blood count. Which of the following medications is most appropriate to administer for this patients cough? Aspirin Dextromethorphan Penicillin Prednisone

Correct Answer ( B ) Explanation: Based on the constellation of findings, this patient most likely has acute bronchitis. Acute bronchitis is a self-limited inflammation of the bronchi and typically presents with mucopurulent cough for more than 5 days. Chest X-ray will show thickening of the bronchial walls in the lower lobes and auscultation of the lungs usually reveals wheezing and rhonchi. Acute bronchitis is most commonly caused by viruses (eg, respiratory syncytial virus, adenovirus, rhinovirus, influenza). Most cases of acute bronchitis are self-limited and resolve on their own within a few weeks. For the patient's cough, over-the-counter medications, such as dextromethorphan or guaifenesin can be offered. Although the benefits of these medications for symptom improvement in patients with acute bronchitis are uncertain, multiple clinical practice guidelines suggest that offering medications for symptom relief may help reduce requests for antibiotics.

A 47-year-old woman presents with several months of a persistent cough that recently resulted in hemoptysis. A review of system also confirms episodes of diarrhea, rash, and flushing. Physical exam is normal with the exception of focal wheezing over the right upper lobe. Which of the following is the most likely diagnosis? Alpha-1-antitrypsin deficiency Carcinoid tumor Pulmonary hypertension Pulmonary tuberculosis

Correct Answer ( B ) Explanation: Bronchial carcinoid tumors commonly cause persistent coughing with hemoptysis and focal wheezing. As these tumors begin to secrete hormonal mediators, they may cause carcinoid syndrome during which symptoms can include diarrhea, flushing, head and neck edema, bronchospasm, or hives. Specific manifestations depend on the type of hormone released, as this can vary. Carcinoid tumors should also be suspected in patients with recurrent pneumonia. Most diagnosed carcinoid tumors grow in the central bronchi. Peripherally-located bronchial carcinoid tumors are rare and usually do not produce symptoms. Suspicion of a bronchial carcinoid tumor should prompt work-up with imaging. A CT scan may help localize the lesion and assess growth over time. Octreotide scintigraphy can also aid in localization. Fiberoptic bronchoscopy usually allows for biopsy in centrally-located tumors. However, biopsy is often complicated by bleeding, as these tumors are generally very well vascularized. Though bronchial carcinoid tumors rarely metastasize, they commonly compromise respiratory function through bleeding and airway obstruction. These tumors are often resistant to radiation and chemotherapy. Fortunately, surgical excision generally gives a favorable prognosis. Alpha-1-antitypsin deficiency (A) may cause respiratory symptoms seen in this patient (persistent coughing, wheezing, and shortness of breath), however it does not classically cause the symptoms of carcinoid syndrome. Pulmonary hypertension (C) is less likely to cause wheezing or hemoptysis and more likely to present with dyspnea on exertion, anginal pain, or even syncope. Physical exam findings may not include wheezing, but rather jugular venous distension, edema, and an accentuated pulmonic valve component on auscultation. Pulmonary tuberculosis (D) may cause a persistent cough and hemoptysis, but is not usually associated with diarrhea, rashes, or afebrile flushing.

A 55-year-old man presents complaining of a 2-day history of cough, fatigue and fever which came on suddenly. He denies chronic medical problems and takes no medications. He does not smoke. He is ill-appearing, though in no distress. His temperature is 101.6°F; pulse 112; BP 122/78; RR 24; and pulse oximetry 92% on room air. Physical exam is remarkable for rales along the right side of his lung fields. What is the most appropriate next step in evaluating his condition? AAcid-fast bacillus smear BChest X-ray CComplete blood count DSputum Gram stain and culture

Correct Answer ( B ) Explanation: Chest X-ray is the most appropriate next step in the evaluation of this patient who has a history and physical exam suggestive of community-acquired bacterial pneumonia (CAP). It is the most common infectious cause of death in the United States. Patients may present with productive cough, fever, shortness of breath, and sometimes pleuritic chest pain. Symptoms may came on suddenly. Clinical findings include tachypnea, tachycardia, fever or hypothermia, and altered breath sounds such as the auscultation of rhonchi or rales, dullness, increased fremitus, or egophony.

You evaluate a 65-year-old patient for shortness of breath and note on exam decreased breath sounds at the left lung base. You are suspicious of a small pleural effusion. In which of the following views on the chest radiograph is the small pleural effusion most likely to be detected? Lateral Lateral decubitus left side down Lateral decubitus right side down Posterior-anterior (PA)

Correct Answer ( B ) Explanation: Classic physical signs of a pleural effusion include diminished breath sounds, dullness to percussion, decreased tactile fremitus, and occasionally a localized pleural friction rub. Chest radiograph confirms the suspicion of pleural effusion. The classic radiographic appearance of a pleural effusion is blunting of the costophrenic angle on the upright chest radiograph Pleural Effusion Transudate: CHF (most common) Exudate: infection > malignancy, PE ↓ Breath sounds + dull percussion + ↓ tactile fremitus CXR: blunting of the costophrenic angle

Which of the following best describes cor pulmonale? Altered structure and function of the left ventricle Altered structure and function of the right ventricle Congenital heart disease Right-sided heart disease due to left-sided heart disease

Correct Answer ( B ) Explanation: Cor pulmonale is the result of pulmonary hypertension associated with diseases of the lung, upper airway, pulmonary vasculature or chest wall. The disease presents as altered structure and function of the right ventricle. Altered structure in cor pulmonale includes both hypertrophy and dilation. Patients present with dyspnea, angina or syncope on exertion, lethargy, and fatigue. Physical exam findings are related to both right-sided heart disease and pulmonary hypertension. They include peripheral edema, elevated jugular venous pressure with prominent v-wave, increased or narrowly split second heart sound and left holosystolic heart murmur at the left lower sternal border. Diagnostic testing such as ECG, echocardiogram and chest X-ray may all be abnormal in patients with cor pulmonale. Treatment is aimed at the underlying cause.

A 16-month-old boy and his adoptive parents present to your office with a complaint of cough and concern about the patient's weight. The parents indicate that they don't have much information about the patient's birth parents and since adopting him four months ago they have noticed daily shortness of breath, cough and wheezing. He is also a fussy eater and is in the second percentile of weight on the growth chart. Which lab finding is most consistent with the diagnosis? Elevated exhaled nitric oxide testing Elevated sweat chloride Elevated white blood cell count Positive skin allergy test

Correct Answer ( B ) Explanation: Cystic fibrosis (CF) is an autosomal recessive disease that occurs predominantly in Caucasians. Multiple organ systems are affected because of a mutation in the cystic fibrosis transmembrane conductance regulator protein, a regulatory protein found in exocrine tissues. Of the patients who survive the neonatal period, 90% have pulmonary complications. Historically patients were diagnosed with cystic fibrosis after presenting with symptoms. Now newborn screening allows for detection of the disease before patients become symptomatic. Clinical presentation includes meconium ileus, failure to thrive and respiratory symptoms. Patients with CF have a shortened lifespan and end-stage lung disease is generally the cause of death. Clinical symptoms, a positive newborn screening or a sibling with the disease are reasons to test for CF. Initial evaluation is with sweat chloride testing and an elevated result should be referred for DNA testing to detect CF-related mutations.

A mother who is at 31 weeks gestation is at high risk of preterm delivery. Which of the following is given prenatally to prevent the risk of infant respiratory distress syndrome? Albumin Betamethasone Phosphatidylglycerol Sphingomyelin

Correct Answer ( B ) Explanation: Glucocorticoids, most commonly betamethasone, are administered to help prevent infant respiratory distress syndrome (hyaline membrane disease) in mothers with premature deliveries. The action of steroids increase the production of surfactant, the key substance which is lacking in newborns who suffer from this condition. Several organizations recommend treatment in women at risk of delivering before 34 weeks gestation. Lung maturity may be tested in women >30 weeks gestation. It is accomplished through the use of amniocentesis. Fluid samples are then sent for different laboratory studies, including the lecithin-to-sphingomyelin ratio, the surfactant-to-albumin ratio and phosphatidylglycerol levels. Albumin (A), phosphatidylglycerol (C) and sphingomyelin (D) are laboratory markers used in the detection of poor lung maturity, not in the prevention of infant respiratory distress syndrome.

What finding on high resolution computed tomographic imaging of the chest is most consistent with idiopathic pulmonary fibrosis? Ground glass patches Honeycombing Non-caseating granulomas Solitary nodule

Correct Answer ( B ) Explanation: Idiopathic pulmonary fibrosis (IPF) is a type of fibrosing interstitial pneumonia of unknown cause. It is characterised by a chronic nonproductive cough and gradual exertional dyspnea that usually develops over several months. IPF is most common in men, aged 60-70 years, with a history of cigarette smoking. The pathogenesis of IPF is likely related to epithelial cell damage and improper repair leading to chronic and progressive symptoms. Medical treatment for IPF has proven difficult, making for a poor prognosis, although nintedanib and pirfenidone seem to delay disease progression. This treatment coupled with supportive care, such as supplemental oxygen, can help maintain a normal level of activity for patients. Since treatment is often delayed by misdiagnosis, it is vital that providers perform a thorough physical exam which often reveals bibasilar crackles and finger clubbing as the disease progresses. Additionally, imaging with high resolution CT reveals structural changes to the lung parenchyma, such as traction bronchiectasis or honeycombing, which refers to the appearance of clusters of cystic airspaces in the lungs. These findings are characteristic of IPF, and in the setting of the appropriate clinical picture, are enough to confirm a diagnosis.

A 76-year-old man with a 30-year pack history of smoking tobacco presents with a chief complaint of increasing dyspnea on exertion over the past 7 months. What aspect of his history and physical examination would suggest idiopathic pulmonary fibrosis, rather than another diagnosis, as the source of his dyspnea? Chest pain Digital clubbing Joint pain Productive coughing

Correct Answer ( B ) Explanation: Idiopathic pulmonary fibrosis (IPF) is a type of fibrosing interstitial pneumonia that most commonly occurs in men over 50 years of age. Although it is linked to cigarette smoking and has been shown to occur in families, its exact cause is unknown. The pathogenesis of IPF is likely related to epithelial cell damage and improper repair leading to chronic and progressive symptoms. The most characteristic symptoms of IPF are a chronic nonproductive cough and gradual exertional dyspnea that usually develops over several months. Medical treatment for IPF has proven difficult, making for a poor prognosis, although nintedanib and pirfenidone seem to delay disease progression. This treatment coupled with supportive care, such as supplemental oxygen, can help maintain a normal level of activity for patients. Since treatment is often delayed by misdiagnosis, it is vital that providers perform a thorough physical exam and order appropriate images. Imaging on high resolution CT reveals structural changes to the lung parenchyma, such as traction bronchiectasis or honeycombing, which refers to the appearance of clusters of cystic airspaces in the lungs. While physical exam may be unremarkable during early disease, it often reveals bibasilar crackles and digital clubbing as the disease progresses, and this clinical presentation along with typical findings on high resolution CT is enough to establish diagnosis.

Which of the following oxygen delivery methods provides the highest fraction of inspired oxygen? Nasal cannula Non-rebreather mask Simple face mask Venturi mask

Correct Answer ( B ) Explanation: Many emergency department patients require supplemental oxygen. Several methods of oxygen delivery are available which provide varying concentrations of oxygen. The choice of method depends on on the patient's clinical condition and oxygen requirement. Nasal cannulae deliver oxygen via prongs inserted into the patient's nostrils, and are capable of oxygen flow rates of 2-5 L per minute, resulting in a FiO2 of 20-50%. A simple face mask can deliver oxygen at flow rates of 6-10 L/min for an FiO2 of 40-60%, and is useful in patients who find nasal prongs irritating, have epistaxis, or other reasons nasal prongs are problematic. Flow rates are determined by the patient's respiratory rate and tidal volume. A venturi mask mixes oxygen with room air at a settable concentration, and can deliver accurate and constant flow rates up to 40% FiO2. A venturi mask is helpful in situations where it is desirable to avoid excessive oxygen administration, such as in a patient with chronic obstructive pulmonary disease. Finally, a non-rebreather mask uses an oxygen reservoir and is capable of delivering the highest concentration of oxygen. Oxygen flow rates approach 10-15 L/minute, delivering an FiO2 of up to 100%. A non-rebreather mask must be secured tightly to the face, and can be uncomfortable for patients. Patients requiring intubation are typically preoxygenated using a nonrebreather mask in order to optimize oxygenation. Noninvasive positive pressure ventilation via continuous positive airway pressure (CPAP) and bilevel positive airway pressure (BiPAP) are also useful in optimizing oxygenation prior to intubation, especially if adequate oxygen saturations cannot be achieved using conventional means.

A 45-year-old patient with newly diagnosed diabetes mellitus type 2 presents to your office for her annual exam. She has had her hepatitis B vaccination, but wants to know if she needs any additional vaccinations because of her new diagnosis. Which of the following is the most appropriate next step in her management? Administer annual influenza vaccine only Administer pneumococcus and annual influenza vaccines Administer pneumonia prophylaxis with trimethoprim-sulfamethoxazole The patient does not need any additional vaccines since she is up to date

Correct Answer ( B ) Explanation: Patients with diabetes mellitus require regular monitoring and health maintenance to prevent diabetes-related complications. Health maintenance for these patients includes three vaccinations: annual influenza, pneumococcus ​(repeated at age 65 if given prior to that age) and the hepatitis B three dose series. Patients with diabetes mellitus require annual foot, dental and dilated eye examinations, blood pressure monitoring, and smoking cessation counseling. Upon diagnosis, a serum creatinine should be drawn. Annual fasting serum lipids and urinary albumin-to-creatinine ratios should be monitored. Hemoglobin A1C should be obtained every 3-6 months with a goal of <7%.

Which of the following is the most common cause of a pleural effusion in developed countries? Bacterial pneumonia Heart failure Malignancy Tuberculosis

Correct Answer ( B ) Explanation: Pleural effusions occur when fluid accumulates between the parietal and visceral pleurae. In a healthy patient, fluid is continually produced by the parietal pleura which reduces friction allowing for smooth lung expansion. If there is overproduction or decreased reabsorption, fluid can accumulate in the potential space. Depending on the volume present, pleural effusions can be clinically silent or cause symptoms, such as dyspnea or chest pain. On physical exam, there will be decreased breath sounds and dullness to percussion. Pleural effusions can be seen on upright chest X-ray when there is more than 150-200 cc of fluid present. Smaller effusions may only be seen on left lateral decubitus films or CT scan. Pleural effusions can be classified as transudates or exudates based on Light criteria. Fluid is an exudate if one or more of the following are present: (1) pleural fluid/serum protein ratio > 0.5, (2) pleural fluid/serum LDH > 0.6, (3) pleural fluid LDH is > 2/3 of the upper limit for serum LDH. Common causes of transudates include congestive heart failure, cirrhosis, and nephrotic syndrome. Common causes of exudates include malignancy, infection, and pulmonary embolism. Overall, the most common cause of a pleural effusion in the Western world is heart failure.

A 36-year-old veterinarian presents with myalgias, dry cough, and severe headache. His vital signs include blood pressure 138/74 mm Hg, heart rate 82 beats/minute, temperature 39°C, and oxygen saturation 94% on room air. He has hepatosplenomegaly on abdominal exam. His chest X-ray shows patchy perihilar infiltrates. What of the following is the most appropriate antibiotic for this patient? Amoxicillin-clavulanate Doxycycline Levofloxacin Trimethoprim-sulfamethoxazole

Correct Answer ( B ) Explanation: Psittacosis is caused by Chlamydia psittaci, an obligate intracellular gram-negative organism. It is harbored in avian species making bird owners, veterinarians, and pet-shop employees particularly susceptible to infection. Patients present with high fevers, severe headache, myalgias, nonproductive cough, and hepatosplenomegaly. Chest X-rays show patchy perihilar or lower lobe infiltrates. Patients may have proteinuria and elevated liver transaminases. Diagnosis should be considered in patients with community acquired pneumonia and exposure to birds. The treatment of choice is a 14-21 day course of doxycycline. Complications are uncommon in patients treated with appropriate antibiotics.

A 12-year-old girl presents to the clinic with one day of cough, fever to 102°F, and extreme fatigue. She is awake and alert but appears tired. Her oxygen saturation is 95 percent, and respiratory rate is 15 breaths per minute. Lung auscultation reveals fine crackles in the left upper lobe. Which of the following is treatment of choice? AAmpicillin BAzithromycinCorrect Answer CCefotaxime DClindamycin

Correct Answer ( B ) Explanation: The above patient's constellation of fever, cough, and focal lung findings are concerning for community-acquired pneumonia. The onset of symptoms was acute and severe, and the lung findings are focal, which is suggestive of a typical bacterial etiology. S. pneumoniae is the most frequent cause of "typical" bacterial pneumonia in children of all ages. However, in otherwise healthy children five years and older with CAP who are not ill enough to require hospitalization, M. pneumoniae and C. pneumoniae are the most likely pathogens. A macrolide is the empirical treatment of choice for typical bacterial pneumonia in otherwise healthy children > 5 years old being treated in outpatient setting. Among the macrolide antibiotics, clarithromycin and azithromycin have a more convenient dosing schedule and fewer side effects than erythromycin, but erythromycin is less expensive. Macrolide antibiotics may provide coverage for S. pneumoniae, which is the most frequent typical bacterial pathogen for all age groups. However, approximately 40 to 50 percent of S. pneumoniae isolates are resistant to macrolides. Failure to respond to macrolide therapy may indicate the development of a complication, a macrolide-resistant pathogen, or the need to alter therapy to provide better pneumococcal coverage. Given the significant resistance of S. pneumoniae to macrolides, fluoroquinolones (e.g., levofloxacin, moxifloxacin) are another reasonable alternative for the outpatient treatment of CAP. In addition to their excellent Gram-negative spectrum, the fluoroquinolones are active against a number of the pathogens responsible for CAP, including beta-lactam-susceptible and non-susceptible S. pneumoniae, M. pneumoniae, and C. pneumoniae. However, S. pneumoniae resistant to levofloxacin has been identified

A seven-year-old boy is brought by his mother to the clinic because of coughing. For the past week, he has had a nonproductive cough. On physical examination, vital signs are normal, with erythematous posterior pharynx, and clear breath sounds. Complete blood count is normal. Chest radiograph reveals perihilar infiltrates. Polymerase chain reaction from the boy's nasopharyngeal specimen comes back positive for Mycoplasma pneumoniae. Which of the following is the most appropriate therapy? Amoxicillin Clarithromycin Levofloxacin Oseltamivir

Correct Answer ( B ) Explanation: The boy has a lower respiratory tract infection due to Mycoplasma pneumoniae. The cough caused by M. pneumoniae infection ranges from nonproductive to mildly productive. Wheezing and dyspnea also may occur, although dyspnea is not a common complaint. Chills are common, but rigors are very rare. Additional respiratory symptoms include pharyngitis, rhinorrhea, and ear pain. Compared to those with pyogenic pneumonia, patients with mycoplasma pneumonia tend to have a more gradual onset of symptoms, less respiratory distress, and usually a normal white blood cell count. The Infectious Diseases Society of America (IDSA) suggests serology or polymerase chain reaction (PCR) tests for the laboratory diagnosis of M. pneumoniae. When available, PCR from a nasopharyngeal specimen can be done rapidly, has a high specificity, and is the diagnostic test of choice. Empiric treatment for M. pneumoniae pneumonia often is initiated based on clinical suspicion given the difficulty with definitive diagnosis. Suspected or documented M. pneumoniae lower respiratory tract infections are treated with a macrolide or tetracycline antibiotic, which is consistent with those in the Pediatric Infectious Diseases Society (PIDS) and the Infectious Diseases Society of America (IDSA) guidelines. Among the macrolide antibiotics, clarithromycin and azithromycin have a more convenient dosing schedule and fewer side effects than erythromycin, but erythromycin is less expensive.

A 29-year-old man with a history of HIV presents with shortness of breath and fever. He has a productive cough but denies hemoptysis. You obtain the chest radiograph seen above. Which of the following is true regarding the patient's diagnosis? Elevated LDH is common Spread is by the hematogenous route Steroids should be administered prior to antibiotics if the PaO2 is <80 Trimethoprim-sulfamethoxazole is the treatment

Correct Answer ( B ) Explanation: The chest radiograph demonstrates miliary tuberculosis (TB), or acute disseminated tuberculosis. The term miliary was first used to describe the pathologic lesions seen on radiography that appeared as small millet seeds. Miliary TB occurs when the host is unable to contain a recently acquired or a dormant TB infection. The condition was mostly seen in young children after primary infection but now is more common in the elderly and in persons infected with HIV. Spread of the mycobacteria occurs through the hematogenous route, which leads to the multisystem nature of miliary TB. Clinically, patients develop many of the similar signs and symptoms of active pulmonary TB—fever, weight loss, anorexia, and weakness. Hemoptysis is uncommon. The classic miliary pattern seen in the radiograph is present in approximately 50% of cases. Hyponatremia is sometimes seen from the development of SIADH. Mortality rates are higher than for other forms of TB, which is likely due to a delay in treatment. Elevated LDH (A) commonly occurs in Pneumocystis jiroveci pneumonia. Also with Pneumocystis jiroveci pneumonia, steroids (C) should be administered prior to antibiotics when the PaO2 is <70 or the A-a gradient is >35. Trimethoprim-sulfamethoxazole (D) is the treatment for Pneumocystis jiroveci pneumonia. Treatment of miliary TB is the same as pulmonary TB.

Which of the following treatments is most appropriate for a patient with acutely symptomatic sarcoidosis? Lung transplantation Prednisone Theophylline Tiotropium

Correct Answer ( B ) Explanation: The treatment of pulmonary sarcoidosis is based on the underlying pathology. Essentially, sarcoidosis is the result of an excessive immune system reaction. As such, anti-inflammatory medications are logical modalities. In up to 75% of all patients with pulmonary sarcoidosis, non-steroidal anti-inflammatory medications are all that is required to obtain symptom relief. However, 15% of patient with pulmonary sarcoidosis exhibit more severe symptoms and pulmonary decline. The cornerstone of therapy for these patients is oral or inhaled corticosteroids, usually given in a tapered fashion over a 6-month period. Acute flares can usually be treated successfully with 3 week tapering doses. Interestingly, there is not much concrete data to support the benefit of corticosteroid treatment even though it is widely used.

A 47-year-old woman with diabetes mellitus is found to have latent tuberculosis during a routine employee health purified protein derivative (PPD) test. She should receive supplementation with pyridoxine (vitamin B6) if her drug regimen contains which of the following medications? Ethambutol Isoniazid Pyrazinamide Rifampin

Correct Answer ( B ) Explanation: The use of isoniazid requires supplementation with pyridoxine in high risk patients (eg. those with diabetes, HIV, chronic alcoholism, renal disease or pregnant or breast-feeding women) to prevent the development of peripheral neuropathies during treatment of active or latent tuberculosis. Ethambutol (A), a bacteriostatic anti-tuberculosis drug, is more likely to cause optic neuritis. This can be distinguished with a visual acuity screening and red-green discrimination test, and is usually reversible with discontinuation of the drug. Pyrazinamide (C), a bactericidal anti-tuberculosis drug, is known to cause hyperuricemia, hepatotoxicity, and joint pain. Side effects should be monitored during treatment with uric acid and liver function tests. Rifampin (D), a bactericidal anti-tuberculosis drug, may also cause hepatitis, as well as flu-like symptoms and poor clotting. Monitoring parameters should include liver function tests and complete blood counts. Patients should also be cautioned that this drug can color bodily secretions orange and stain contact lenses.

A 55-year-old man, smoker, presents to the ED with hemoptysis and dyspnea for four weeks. His vital signs are T 37°C, BP 146/76 mm Hg, HR 85 bpm, RR 20 per minute, and oxygen saturation 96% on RA. His lung exam reveals distant breath sounds on the left side. His chest X-ray is shown above. What is the most likely cause of his hemoptysis? ABronchitis BLung cancerCorrect Answer CPneumonia DPulmonary embolism

Correct Answer ( B ) Explanation: This patient has a lung mass that has eroded into a pulmonary vessel. The pulmonary blood supply is under low pressure, so bleeding can present as blood-tinged sputum or gross hemoptysis. Massive hemoptysis is caused by hemorrhage from a bronchial vessel in the majority of cases because bronchial vessels are under systemic blood pressure. Some 80%-90% of patients with cancer as the cause of hemoptysis will have abnormalities on the chest X-ray. Those with normal chest x-rays with a high suspicion for an underlying neoplasm should undergo a chest CT scan and have an outpatient fiber-optic bronchoscopy.

A 19-year-old man presents with a sore throat and difficulty swallowing. He has had 4 days of worsening sore throat and fever but today he was unable to swallow any liquids. Physical examination reveals a muffled voice, difficulty tolerating saliva, and minimal pharyngeal erythema with a midline uvula. He also has tenderness over the hyoid bone. Which of the following managements is most likely indicated? Inhaled racemic epinephrine and discharge home Intravenous antibiotics and admission Intravenous corticosteroids and discharge home Needle aspiration of peritonsilar area

Correct Answer ( B ) Explanation: This patient presents with symptoms and signs concerning for epiglottitis and should have antibiotics started and admission for further monitoring. Epiglottitis is a localized cellulitis of the supraglottic structures including the aryepiglottic folds, arytenoids, lingual tonsils, epiglottis and base of the tongue. It may be caused by viral or bacterial agents (Haemophilus influenzae type b is the most common). Although incidence in pediatric populations has decreased with the H. influenzae vaccine, it is becoming more common in adults and can lead to rapid airway obstruction. Patients usually present with dysphagia, odynophagia and sore throat. Pain may be severe in relation to physical exam findings. Additionally, patients frequently complain of a muffled voice. Fever is present in half of cases. Physical examination typically reveals mild inflammation in the oropharynx and may reveal tenderness to the neck over the hyoid bone. Epiglottitis is a clinical diagnosis but may be aided by plain radiographs of the neck, which shows edema of the epiglottis (thumb sign). An epiglottic width greater than 8 mm or an aryepiglottic fold greater than 7 mm suggests epiglottitis. Definitive diagnosis can be made with indirect laryngoscopy. Management in patients focuses on protecting the airway when necessary starting IV antibiotics (ceftriaxone and ampicillin-sulbactam are first-line) and admission for monitoring. Supportive care with hydration is important as well

A 30-year old woman presents with shortness of breath. She has a history of asthma since childhood. She says that she uses her rescue inhaler 3 days each week. On average, she wakes up in the middle of the night with symptoms approximately three times a month. Her vital signs are BP 120/76, HR 112, RR 26, and oxygen saturation 92% on room air. The patient appears to be in mild distress with intercostal retractions present, and respiratory exam is remarkable for diffuse wheezing. This patient is diagnosed with an acute asthma exacerbation. Which of the following best classifies this patient's asthma? Intermittent Mild persistent Moderate persistent Severe persistent

Correct Answer ( B ) Explanation: This patient's asthma is classified as mild persistent. Symptoms of asthma most frequently include shortness of breath and wheezing, but patients may also complain of cough and chest tightness. Mild persistent asthma is classified as symptoms more than twice per week but not daily, nocturnal symptoms three or four times each month, exacerbations causing minor limitations in activity and sleep, and the use of an inhaled short-acting beta-2-agonist more than twice a week but not daily. These patients are generally treated with a low-dose inhaled corticosteroid with a short acting beta-agonist as needed.

A 28-year-old man presents to the Emergency Department with dyspnea. On an anterior-posterior chest X-ray, you notice a visceral pleural line with a radiolucent area devoid of vascular and pulmonary markings on the right side only. What is the most likely diagnosis? Adenocarcinoma Pneumonia Pneumothorax Pulmonary embolism

Correct Answer ( C ) Explanation: A pneumothorax is characterized by unilateral findings of a visceral pleural line with no vascular pulmonary markings in between this line in the chest wall on chest X-ray. A pneumothorax is a collection of air in the pleural space between the lung and the chest wall. The cause may be spontaneous, iatrogenic, or traumatic. The highest frequency of spontaneous pneumothorax occurs in tall, thin males aged 10 to 30 years. The usual patient presentation is with symptoms of dyspnea and chest pain. Physical exam findings are decreased or absent breath sounds unilaterally, as well as unilateral chest expansion and hyperresonance. Chest X-ray confirms the presence of pleural air, with a visceral pleural line evident. A small pneumothorax may resolve spontaneously, but most require placement of a chest tube. A tension pneumothorax is a medical emergency. This is often due to a chest wound or laceration that allows inspiration of air but not expiration. This causes mediastinal shift to the contralateral side and cardiovascular compromise. A large-bore needle is inserted into the chest to allow expiration of air, before placement of a chest tube.

A 22-year-old woman presents with dyspnea. She has a history of asthma and noted increased difficulty breathing starting yesterday. She says she has been using her albuterol inhaler every 15 minutes for the last four hours without relief. What laboratory abnormality is likely to be found in this patient? Hypocalcemia Hypoglycemia Hypokalemia Hyponatremia

Correct Answer ( C ) Explanation: Albuterol, a beta-2-adrenergic agonist, is a widely used medication in the treatment of asthma and chronic obstructive pulmonary disease. Beta-2-adrenergic agonists work selectively on the beta-2 receptors on bronchial smooth muscle resulting in bronchodilation. The most common side effects include tremor and tachycardia. Overuse of these medications can also result in hypokalemia due to increased activity of Na-K-ATPase pumps in skeletal muscle which drive potassium intracellularly. Serum potassium levels can be decreased by 0.5 to 1.5 mEq/L. While this is unlikely to result in clinically significant hypokalemia, it can be useful in patients who present with acute hyperkalemia. While not recommended as monotherapy in the treatment of hyperkalemia, it has been shown to have additive effects when used in conjunction with insulin and glucose in transiently lowering potassium levels.

An unimmunized 7-year-old boy without any previous medical history presents with fever and cough. The parents report that he has been coughing for 2 weeks. The patient has sudden fits of coughing followed sometimes by post-tussive emesis. Which of the following is an appropriate plan? Albuterol nebulizer Corticosteroid therapy Macrolide antibiotic Respiratory viral panel

Correct Answer ( C ) Explanation: Although a vaccine for pertussis was developed in the 1940s, it remains a cause of significant respiratory illness in the US each year. Decreasing immunity in adults has contributed to persistent annual cases of this disease. The disease occurs most commonly in children under the age of one who have not received the entire vaccine series and nonimmunized children. Pertussis is caused by Bordatella spp. which specifically adheres to the ciliated epithelial cells of the respiratory tract. There are three phases to the illness. The catarrhal phase lasts 1-2 weeks after an incubation period of 7-10 days. Patients are most infectious during this phase, the symptoms of which are indistinguishable from an average upper respiratory infection. Phase two is the paroxysmal phase characterized by the resolution of fever and increased cough. For 2-4 weeks, patients have paroxysms of the classic "staccato" cough. Patients cough in fits followed sometimes by the "whoop" when they forcefully inhale. Post-tussive emesis, syncope and brief apneic episodes may occur. The convalescent third phase lasts for weeks to months and is characterized by a residual cough. Antibiotic therapy with a macrolide is indicated to decrease the infectivity of patients. The use of antibiotics does not decrease the duration of illness or severity of illness. The primary treatment is supportive care. Erythromycin is contraindicated for infants < 1 month due to increased risk for pyloric stenosis. Infants < 1 month old should receive azithromycin.

Which of the following complications can be prevented by simultaneously administering pyridoxine and isoniazid in a patient with tuberculosis exposure? Color blindness Hepatitis Peripheral neuropathy Renal failure

Correct Answer ( C ) Explanation: Isoniazid (INH) inhibits the enzyme responsible for the conversion of pyridoxine (vitamin B6) to one of its active metabolites, pyridoxal phosphate (PLP). This depletion of vitamin B6 may lead to complications such as peripheral neuropathy and seizures. Therefore, vitamin B6 should be administered concomitantly to patients taking isoniazid. PLP is also a coenzyme required for the synthesis of gamma-aminobutyric acid (GABA), an inhibitory neurotransmitter. Decreased GABA formation in the setting of vitamin B6 deficiency may also contribute to seizures. Color blindness (A) is not a complication of INH. However, another commonly used drug in TB, ethambutol, is associated with retrobulbar neuritis and red-green color blindness. INH is metabolized by the liver and gets converted to an ammonium molecule that can lead to hepatotoxicity (B). However, this is not affected by vitamin B6 supplementation. Renal failure (D) is a complication of pyridoxine overdose.

Which of the following statements is true concerning aspiration pneumonitis? Antibiotics should be administered early in the course Corticosteroid administration decreases the incidence of associated lung injury It is caused by an inflammatory chemical injury The ABG is diagnostic

Correct Answer ( C ) Explanation: Aspiration pneumonitis is an inflammatory chemical injury of the tracheobronchial tree and pulmonary parenchyma produced from the inhalation of regurgitated gastric contents. Aspiration pneumonitis can lead to aspiration pneumonia due to the breakdown of the pulmonary defense mechanisms caused by the chemical irritation. Aspiration pneumonitis is usually associated with depressed level of consciousness that allows for regurgitation of gastric contents and inhibits the protective upper airway reflexes from preventing aspiration. The classic patient has a depressed mental status due to recreational or therapeutic drugs and regurgitates a large volume of gastric contents. The elderly and those patients with a history of stroke are also at increased risk for aspiration pneumonitis. Clinically, patients may have minor symptoms (nonproductive cough and tachypnea). With larger aspirations, patients may develop tracheobronchitis with bronchospasm, bloody or frothy sputum, and respiratory distress. The chest radiograph usually shows unilateral focal or patchy consolidations in the dependent lung segments.

A 60-year-old man with a past medical history of tobacco use presents to clinic with a chief complaint of cough. The cough has been present for five months, and he now has difficulty breathing when walking to his mailbox. He also had a "coughing fit" similar to this last year lasting three to four months. What is the most likely diagnosis? Acute bronchitis Asthma Chronic bronchitis Emphysema

Correct Answer ( C ) Explanation: Chronic bronchitis is a form of chronic obstructive pulmonary disease that is defined as a chronic productive cough for at least three months in at least two successive years. In chronic bronchitis, the bronchi become irreversibly inflamed causing mucus hypersecretion and coughing. It is described as the "blue bloater" as patients are commonly overweight and cyanotic. Symptoms include chronic cough, purulent sputum, and dyspnea, particularly on exertion. Physical examination in a patient with chronic bronchitis reveals decreased breath sounds, increased resonance upon percussion of the lung fields, and use of accessory muscles to breathe. Treatment includes smoking cessation, lifestyle changes with pulmonary rehabilitation, inhaled bronchodilators, and inhaled steroids

You are seeing an elderly man with new onset of peripheral edema, head fullness and neck venous engorgement. Initial testing shows normal left heart function. You suspect cor pulmonale. Which of the following tests is the most accurate in confirming this diagnosis? Cardiac magnetic resonance imaging Electrocardiography Right heart catheterization Ultrafast ECG-gated computed tomography

Correct Answer ( C ) Explanation: Cor pulmonale is defined as an alteration in the structure and function of the right ventricle caused by a primary disorder of the respiratory system. Although the most common cause of right heart failure is left-sided heart disease, cor pulmonale is right heart dysfunction due to a lung, and not heart, problem. Just like systemic hypertension causes changes in left ventricular function, pulmonary hypertension causes changes in right ventricular function. This major underlying pathology is largely due to some kind of pulmonary vascular bed compromise, which can be primary pulmonary hypertension or thromboembolic disease, but more commonly anatomic compromise (COPD, interstitial lung disease and rheumatologic or connective tissue/collagen vascular disorders) or vasoconstrictive compromise (chronic hypoxic states and acidemia). When evaluating a patient with right heart failure, cor pulmonale is considered if pulmonary pathology is causative. However, if the etiologic evaluation is void of a pulmonary source, then the diagnosis of cor pulmonale cannot be made. In this situation, the clinician then tries to pinpoint a cardiac or blood disorder. Even when a pulmonary source is found to be a cause of right side heart failure, it is equally important to determine if there is such a coexisting non-pulmonary cause, such as increased blood viscosity, atrial and ventricular defects, congenital heart disease, cardiomyopathies and constrictive pericarditis. The general approach to evaluating a patient with suspected cor pulmonale begins with routine lab testing, chest radiography and electrocardiography (ECG). Further investigation of underlying pulmonary pathology is then accomplished via pulmonary function testing, ventilation/perfusion (V/Q) scanning and chest computed tomography. Right heart catheterization is the most accurate but invasive test to confirm the diagnosis of cor pulmonale.

Which of the following physiologic responses would occur after application of noninvasive positive pressure ventilation in a patient presenting with an acute exacerbation of chronic obstructive pulmonary disease? Increased afterload Increased alveolar dead space Increased tidal volumes Increased venous return

Correct Answer ( C ) Explanation: Noninvasive positive pressure ventilation applies a consistently positive airway pressure to increase laminar flow. This leads to airway stenting, elimination of dead space through alveolar recruitment, and an increase in tidal volumes and minute ventilation. The beneficial effects of positive pressure ventilation are not only realized in the pulmonary system but also in the cardiovascular system. Patients with pulmonary edema from decompensated heart failure benefit from the increased intrathoracic pressure which decreases venous return and increases left heart output and thus decreases afterload. Increased afterload (A) is incorrect because the increased intrathoracic pressure increases left heart output which decreases afterload. Increased alveolar dead space (B) is incorrect because airway stenting recruits alveoli and decreases the alveolar dead space. Increased venous return (D) is incorrect because the increased intrathoracic pressure actually decreases the venous return.

You are treating a 50-year-old coal-miner's hypertension with lisinopril. He has been complaining of 3-months of progressive dyspnea. You order a chest radiograph which shows bilateral upper lobe honeycombing. A high resolution computed tomogram shows multiple small, round opacities only in the upper lobes. The lower lobes appear normal. Which of the following is the most likely diagnosis? Goodpasture's syndrome Granulomatosis with polyangiitis (GPA) Pneumoconiosis Scleroderma

Correct Answer ( C ) Explanation: Pneumoconiosis is an occupational respiratory disease due to inhalation of inorganic dusts. It is common in miners. In 2010, it resulted in 125,000 deaths in the US. History is key in determining the probable agent. A common form is Coalworker's pneumoconiosis, also known as miner's lung or black lung, and which is due to chronic inhalation of coal and carbon particles. Machinists, especially grinders, and pottery makers, can develop the silicon particle form called silicosis. Construction workers, especially boat builders, can suffer from asbestosis. Clinically, the diagnosis is made in a patient with dyspnea and the classic occupational exposures as above. Radiography may reveal small cystic radiolucencies described as honeycombing. High-resolution computed tomography offers better evaluation, and typically shows small, round opacities which denote inflammatory areas of dust-laden macrophages and fibrosis, as is quite common in the carbon deposits of coalworker's pneumoconiosis. Interestingly, the upper lobes are more affected in miner's lung and silicosis, while the lower lobes are more affected in asbestosis. Progressive pulmonary fibrosis, with resultant restrictive lung disease pathology and pulmonary function test results, typically occurs. Other than removing oneself from the offending agent and dusty environment, treatment is mainly supportive Goodpasture's syndrome (A) is an interstitial lung disease characterized by alveolar hemorrhage and glomerulonephritis. Scleroderma (D) is a collagen vascular disease whose pulmonary findings include lower lobe fibrosis and pulmonary hypertension. Upper lobe pathology is uncommon. Granulomatosis with polyangiitis (GPA) (B) is a vasculitic interstitial lung disease whose imaging reveals necrotizing granulomas

A 33-year-old man presents with five days of gradual onset nonproductive cough, fatigue, and fever. He also notes a 15 pound weight loss over the last month. He is tachypneic with a heart rate of 105 beats/minute, temperature of 38.2oC, and an oxygen saturation of 89% on room air. On examination, white plaques are noted on his tongue and his lungs are clear on auscultation. His chest X-ray is shown above. Which of the following is the most likely causative agent? Influenza A Klebsiella pneumoniae Pneumocystis jiroveci Streptococcus pneumoniae

Correct Answer ( C ) Explanation: Pneumocystis jiroveci (formerly carinii) pneumonia (PCP) is an opportunistic fungal pathogen which primarily affects immunocompromised patients including those with HIV/AIDS, as well as cancer and organ transplant patients on immunosuppressants. PCP is one of the most common AIDS-defining opportunistic infections. Patients present with gradual onset of nonproductive cough, fever, dyspnea, and decreased exercise tolerance. Patients with previously undiagnosed HIV/AIDS may also present with symptoms, such as weight loss and oral candidiasis. Lung auscultation is often normal although rales or rhonchi may be present. Chest X-ray shows bilateral diffuse interstitial perihilar infiltrates that extend in a "bat-wing" pattern, although atypical findings, such as a normal chest X-ray or apical infiltrates can be seen. Spontaneous pneumothorax is also seen in about 10% of patients. Arterial blood gases are often abnormal with decreased pO2 and respiratory alkalosis. LDH is increased, with the degree of elevation used as a marker for a worse prognosis. In HIV patients, CD4 counts are typically < 200 cells/mm3. Intravenous trimethoprim-sulfamethoxazole is the antibiotic of choice. Steroids are indicated in patients with moderate to severe disease as indicated by pO2 < 70 mmHg. They have been found to decrease mortality and accelerate recovery.

Which of the following is the most common opportunistic respiratory infection in patients with acquired immunodeficiency syndrome? Cytomegalovirus Mycobacterium tuberculosis Pneumocystis jiroveci Streptococcus pneumoniae

Correct Answer ( C ) Explanation: Pneumocystis jiroveci is the most common opportunistic infection in patients with acquired immunodeficiency syndrome (AIDS). Pneumocystis jiroveci pneumonia (PCP) typically occurs in human immunodeficiency virus (HIV)-positive patients with a CD4 < 200 cells/microL who are not on potent antiretroviral therapy or PCP prophylaxis. PCP can also be seen in patients with cancer, stem cell or solid organ transplantation, primary immunodeficiencies, or severe malnutrition. P. jiroveci is primarily transmitted via the airborne route. The clinical manifestations of PCP include fever, nonproductive cough, and dyspnea. Fever and tachypnea are the most common physical exam findings. Rhonchi and crackles may be heard on chest exam. The most common abnormalities seen on chest radiograph are diffuse, bilateral, interstitial, or alveolar infiltrates. The gold standard for diagnosing PCP is immunofluorescent staining using fluorescein-labeled monoclonal antibodies . Empiric therapy should be initiated in patients who are acutely ill and in whom there is a high clinical suspicion. Trimethoprim-sulfamethoxazole (TMP-SMX) is the mainstay treatment for PCP. Patients with mild to moderate disease, based on respiratory status, can be managed with oral TMP-SMX. Patients with severe disease should receive intravenous therapy. Trimethoprim-dapsone is an alternative treatment for patients with a sulfa allergy. Adjunctive corticosteroids are recommended in patients with moderate-to-severe disease. TMP-SMX is also the most effective agent for PCP prophylaxis.

Which of the following diagnoses should be considered when an unexplained isolated pleural effusion is found on chest radiograph? Bacterial pneumonia Congestive heart failure Pulmonary embolism Viral pleuritis

Correct Answer ( C ) Explanation: Pulmonary embolism is the most commonly overlooked condition in the workup of pleural effusion. Any patient with an unexplained pleural effusion should be evaluated for possible pulmonary embolism using CT or V/Q scanning. Pleural effusion resulting from pulmonary embolism usually occupies less than 33% of the hemithorax but is accompanied by a greater degree of dyspnea than would be expected. The classic radiographic appearance of a pleural effusion is blunting of the costophrenic angle on the upright chest radiograph. On the AP or PA projection, a volume of 250-500 mL of fluid is required before the effusion can be appreciated. A lesser amount may be visible on the lateral projection in the posterior costophrenic gutter

76-year-old woman with a history of chronic obstructive pulmonary disease presents to your office in December with complaints of fever, muscle aches, headache and malaise that started yesterday. Which of the following is the most appropriate next step in management? Advise supportive care measures including acetaminophen Begin a course of azithromycin Begin a course of oseltamivir Begin a course of prednisone

Correct Answer ( C ) Explanation: Seasonal influenza is caused by influenza A or B viruses and results in acute respiratory illness. Clinical presentation includes fever, myalgia, headache and malaise and high-risk patients can experience serious complications including death. Antiviral therapy can shorten the duration of illness and viral shedding and decrease the severity of symptoms. Patients with symptoms that began 24-48 hours prior to presentation should be evaluated for being at high risk of complications and those at high risk should start a course of oseltamivir. Groups considered to be at high risk include adults 65 years and older, children 2 years and younger, immunocompromised patients, pregnant women, residents of nursing homes or chronic care facilities, and individuals with certain chronic illnesses including chronic pulmonary disease.

Which of the following is the most common electrocardiogram finding in an acute pulmonary embolism? Right bundle branch block S wave in lead I, Q wave in III, inverted T wave in III (S1Q3T3) Sinus tachycardia T wave inversions in leads V1-V4

Correct Answer ( C ) Explanation: Sinus tachycardia is the most common ECG abnormality seen in a pulmonary embolism. It is seen in almost half of all cases. A pulmonary embolism most commonly occurs when a deep venous thrombosis (DVT) embolizes from the lower extremities and travels to the pulmonary vasculature. This leads to a ventilation-perfusion mismatch in the lungs resulting in hypoxia. Patients with a pulmonary embolism present with dyspnea, pleuritic chest pain, and tachycardia. Risk factors include malignancy, immobilization, recent surgery, and genetic hypercoagulable states. In cases with a large clot burden, acute pulmonary hypertension occurs and a right heart strain pattern will be seen on ECG. A massive pulmonary embolism is defined as a large clot burden causing obstructive shock. Treatment includes systemic thrombolytics, catheter-directed thrombolytics, and rarely embolectomy.

Which of the following is the most common location of aspirated foreign bodies in children? Left lower bronchus Left main bronchus Right main bronchus Trachea

Correct Answer ( C ) Explanation: The most common location of aspirated foreign bodies in children is the right main bronchus, followed by the left main bronchus, and the trachea. Foreign body aspiration (FBA) into the trachea or bronchi is a potentially life-threatening event because it can obstruct the airway and prevent oxygenation and ventilation. In children younger than two years old, FBA is a frequent cause of mortality and morbidity. The peak incidence of FBA is between 1-2 years of age. Peanuts, seeds, popcorn, and pieces of toys are some of the most commonly aspirated objects by infants and toddlers. Older children are more likely to aspirate non-food items. Round, solid objects (e.g. grapes and hotdogs) are the most dangerous aspirated objects because they can completely occlude the airway. Signs and symptoms depend on the degree of foreign body obstruction. Children who present with severe respiratory distress, stridor, aphonia, and loss of consciousness represent a true medical emergency and require urgent recognition. However, less emergent situation are more common and present with wheezing, cough, and diminished breath sounds. In children or infants who present with complete airway obstruction, dislodgement using back blows or the Heimlich maneuver should be attempted. Oxygen administration and other life support should be provided until rigid bronchoscopy can be performed. Blind sweeps of the mouth should be avoided due to the risk of pushing an object further down the throat. In less emergent situations, a plain chest radiograph may be useful depending on if the object is opaque and the degree of airway obstruction. Normal findings on radiography cannot be used to rule out FBA. All cases of suspected FBA should undergo examination of the tracheobronchial tree with a rigid bronchoscopy. Retained aspirated foreign bodies may lead to bronchiectasis.

A 33-year-old woman comes to the clinic complaining of blurred vision, cough, and fatigue. Her blood pressure is 135/90 mm Hg and temperature is 38.6°C (101.5°F). She has no past medical history. Physical examination shows several subcutaneous nodules on both lower extremities. Ophthalmologic examination shows uveitis. Chest X-ray shows bilateral hilar adenopathy and reticular opacities. Laboratory studies show leukopenia, eosinophilia, and an elevated erythrocyte sedimentation rate. Which of the following is the most likely diagnosis? Asbestosis Cystic fibrosis Sarcoidosis Tuberculosis

Correct Answer ( C ) Explanation: Uveitis, bilateral hilar adenopathy, dry cough, subcutaneous nodules (eg, erythema nodosum), and an elevated erythrocyte sedimentation rate (ESR) most likely suggest sarcoidosis. Sarcoidosis is granulomatous disease that can form nodules in multiple organs (eg, skin, lungs). Patients typically present with fatigue, weight loss, arthritis, dry eyes, blurry vision, and respiratory symptoms (eg, cough, dyspnea). Management usually involves the use of corticosteroids (eg, prednisone). Bilateral hilar adenopathy, seen on chest x-ray, is a classic finding associated with sarcoidosis.

What is the most common cause of croup? Adenovirus Haemophilus influenzae type b Parainfluenza virus Streptococcus spp.

Correct Answer ( C ) Explanation: The parainfluenza virus is the most common cause of croup. Croup is classically associated with a barking, seal-like cough and inspiratory stridor. Radiographs may show subglottic narrowing ("steeple sign") caused by edema. The typical age group is 6 months to 3 years, but the condition can be seen in children up to 5 years. The infection and inflammation are usually self-limiting, and conservative management is recommended. Evidence supports the routine use of corticosteroids in most children with croup. Intervention at an earlier phase of the illness reduces the severity of symptoms and the rates of return to a health care practitioner for additional medical attention, ED visits, and hospital admissions. Many children respond to a single, oral dose of dexamethasone. For those who do not tolerate the oral preparation, nebulized budesonide or intramuscular dexamethasone are reasonable alternatives. Mild disease can be treated with humidified oxygen. Moderate to severe disease should be treated with steroids and nebulized racemic epinephrine Adenovirus (A) is a common cause of URIs, but less common than parainfluenza virus as a cause for croup. Haemophilus influenzae type b (B) is implicated in various infections such as otitis media and epiglottitis, but less common as a cause of croup. Streptococcus spp. (D) infection is not a common cause of croup.

A 64-year-old man presents with a cough and shortness of breath for 2 weeks. His chest X-ray is shown above. Which of the following tests would indicate an infectious process as the cause of the above finding? Pleural fluid to serum LDH ratio < 0.6 Pleural fluid to serum protein ratio < 0.1 Pleural fluid to serum protein ratio > 0.5 Pleural fluid WBC 5,000 cells/mm3

Correct Answer ( C ) Explanation: The patient presents with a large pleural effusion that may be secondary to a pneumonia and a pleural fluid:serum protein ratio > 0.5 indicates the presence of an exudative effusion consistent with an infectious cause. Pleural effusions can result from a number of different pathologies. This includes CHF, malignancy, bacterial pneumonia, pulmonary embolism and pancreatitis. In developing countries, tuberculosis infection is the most common cause of pleural effusions. Pleural effusions can either be transudative (CHF, cirrhosis, pulmonary embolism) or exudative (malignancy, pneumonia, pulmonary embolism, pancreatitis). Patients will often present with dyspnea and pleuritic chest pain and dullness to percussion over the effusion. It is often difficult clinically to determine the cause of the effusion but testing of a sample of the fluid can differentiate exudative from transudative processes. Light's criteria (see table) is typically used for this purpose. If a single criterion is consistent with an exudative process, the fluid is exudative in origin.

A 32-year-old woman presents to your office with a complaint of productive cough that started 6 days ago. She reports occasional wheezing, chest wall tenderness, and has been afebrile since the onset of symptoms. Her husband had similar symptoms and was prescribed azithromycin by his primary care provider. Which of the following is the most appropriate next step in management? Order a chest X-ray Prescribe a course of azithromycin Send a sputum sample for culture Symptomatic treatment

Correct Answer ( D ) Explanation: Acute bronchitis is an inflammation of the bronchi caused by upper respiratory infection. It is a self-limiting condition that is most commonly of viral etiology. Patients with acute bronchitis generally have few systemic symptoms. The most common clinical presentation is a cough lasting up to 10-20 days. The cough may be productive and patients often mistakenly attribute purulent sputum in acute bronchitis with a bacterial infection. The symptoms of acute bronchitis are less severe than those of influenza or pneumonia and help determine diagnosis and course of treatment. Symptomatic treatment with acetaminophen or NSAIDs and other supportive care measures are recommended as initial management of patients with acute bronchitis.

Which of the following organisms is most associated with pneumonia and bullous myringitis? Bordetella pertussis Haemophilus influenzae Streptococcus bovis Streptococcus pneumoniae

Correct Answer ( D ) Explanation: Although bullous myringitis is described as a classic finding in M. pneumoniae infections, it is not specific for mycoplasmal infection and is present in only a few cases. S. pneumoniae infection is most likely the cause of bullous myringitis in the setting of pneumonia. S. pneumoniae is a common cause of community acquired pneumonia. Bullous myringitis is an inflammation of the eardrum in which painful, fluid filled vesicles form. Patients with S. pneumoniae will present with typical signs of pneumonia. The presence of otalgia should prompt evaluation for bullous myringitis. Bullous myringitis will resolve with antibiotics directed at the S. pneumoniae pneumonia. Bullous Myringitis S. pneumoniae Fluid filled vesicles on TM Pneumonia + otalgia Bullous myringitis is not primarily associated with mycoplasma pneumoniae: Bullous myringitis is a presentation of acute otitis media in which bullae are seen on the tympanic membrane. Despite earlier belief that this condition was associated with mycoplasmal infection, more recent evidence indicates that the prevalence of viral, bacterial, or mycoplasmal infection is the same in bullous myringitis as in nonbullous otitis media.

A 58-year-old man presents to your office with complaints of chronic cough and difficulty breathing. He has smoked approximately 2 packs of cigarettes per day since he was 14 years old and has not seen a doctor in many years. You order spirometry testing, which shows airflow limitation (predicted FEV1 45%). Which of the following is the most appropriate therapy? Azithromycin Guaifenesin Prednisone Tiotropium

Correct Answer ( D ) Explanation: Chronic obstructive pulmonary disease (COPD) is a progressive condition characterized by airflow limitation that causes an enhanced inflammatory response in the airways and lungs. Historically COPD was divided into three subtypes: chronic bronchitis, emphysema and asthma. Asthma is no longer grouped with COPD and current definitions of COPD do not differentiate between emphysema and chronic bronchitis. Patients generally have a lengthy smoking history and are in their fifth decade of life. They often have a chronic cough with sputum production. Diagnosis is by pulmonary function tests including spirometry. Treatment is based on the severity of the disease. Long-acting anticholinergics such as tiotropium are the mainstay of therapy.

A 4-year-old patient is brought to the emergency department by his parents after a choking incident that occurred one hour ago. The parents tell you that their son was eating steak, started to laugh and then began choking. Since the incident, he has been coughing and wheezing. Physical exam reveals unilateral diminished breath sounds with auscultation of the lungs. Which of the following is the most appropriate next step in management? Begin a course of antibiotics and steroids Chest X-ray Flexible bronchoscopy Rigid bronchoscopy

Correct Answer ( D ) Explanation: Foreign body aspiration (FBA) can be a life-threatening event and is a common cause of morbidity and mortality in children. The majority of pediatric FBA incidents occur in children less than 3 years of age. Types of foreign bodies that are commonly aspirated by children include peanuts, popcorn, other nuts and seeds, food particles and pieces of toys. FBA is a true medical emergency if there is a complete airway obstruction. Children with cyanosis, altered mental status and severe respiratory distress require immediate medical intervention including life support and rigid bronchoscopy to remove the foreign body. All cases of suspected FBA require that the tracheobronchial tree be examined. Rigid bronchoscopy is used for this examination and is both a diagnostic and management tool for partial and complete obstructions caused by FBA. Chest X-ray (B) may detect FBA if the aspirated object is radioopaque or depending on the severity of the obstruction. Normal findings on a chest X-ray do not rule out FBA since foods are not radioopaque and are commonly aspirated by pediatric patients. Flexible bronchoscopy (C) is often used in children with pneumonia or other respiratory symptoms. Use of flexible bronchoscopy for diagnosis of FBA may dislodge the object and cause a partial obstruction to become complete.

A patient with significant dyspnea presents for evaluation. You order spirometric testing and obtain results consistent with restrictive pulmonary disease. Which of the following findings is most consistent with restrictive pulmonary disease? Alveolar destruction Bronchoconstriction Excess mucus production Parenchymal abnormalities

Correct Answer ( D ) Explanation: Pulmonary diseases can be classified as obstructive and restrictive. The obstructive conditions, asthma, emphysema, chronic bronchitis, bronchiolitis and bronchiectasis, exist in a state of increased airway resistance, increased compliance and decreased parenchymal recoil. These conditions result in a normal or slightly increased forced vital capacity (FVC), and a decreased forced expiratory volume in the first second of expiration (FEV1) to FVC ratio. On the other hand, restrictive conditions result in decreased airway compliance and increased recoil. This leads to a decrease in the FVC and FEV1 but a near normal FEV1/FVC ratio. The restrictive pulmonary conditions include the interstitial lung diseases (ILD) (parenchymal abnormalities) and the chest wall disorders. Interstitial lung disease includes idiopathic interstitial pneumonia (mostly idiopathic pulmonary fibrosis), drug and radiation induced (iatrogenic) pulmonary fibrosis, hypersensitivity pneumonitis, collagen vascular disease, pneumoconiosis and sarcoidosis. Chest wall restrictive lung disease includes pleural disease, severe obesity and some neuromuscular disorders, such as Guillain-Barre syndrome

Which of the following is a risk factor for pulmonary embolism? Chronic obstructive pulmonary disease Cirrhosis Low body mass index Recent abdominal surgery

Correct Answer ( D ) Explanation: Pulmonary embolism (PE) is the obstruction of pulmonary venous vasculature by material that originated elsewhere in the body, most commonly a thrombus. Other source material for pulmonary embolism includes air, fat from long bone fractures, and tumor material from malignancy. The most common source of thromboembolic disease to the lungs is the proximal deep veins of the lower extremities (eg. iliac, femoral, and popliteal veins). Pulmonary embolism has a large spectrum of clinical presentations, ranging from asymptomatic to hemodynamic instability with overt obstructive shock and death. Symptoms of pulmonary embolism may present acutely, subacutely, or even chronically and most commonly include dyspnea with pleuritic chest pain, cough, and hemoptysis. Patients may also have symptoms of deep venous thrombosis, including unilateral leg swelling, tenderness, and palpable cords. Risk factors for PE are similar to those for venous thromboembolic disease and include recent surgical or accidental trauma, immobilization, malignant neoplasm, pregnancy or use of estrogen products, clotting disorders, and central venous catheter or pacemaker placement. Patients with low risk for PE based on risk factors may be screened with a D-dimer level, which is extremely sensitive but nonspecific for pulmonary embolism. Hemodynamically stable patients with higher risk should undergo definitive imaging with computed tomographic pulmonary angiography or, if unable to tolerate contrast dye due to poor renal function, ventilation perfusion scanning. Management depends on the hemodynamic stability of the patient. The cornerstone of therapy in stable patients is immediate and long term anticoagulation. Hemodynamically unstable patients require fibrinolysis or embolectomy.

An obese 34-year-old woman is brought to the Emergency Department with respiratory distress. Two months ago she was in the hospital for knee surgery. Paramedics report an acute onset of dyspnea and pleuritic chest pain. She also complains of a tender thigh on the same side of her knee surgery. She is tachycardic and tachypneic, and mildly hypotensive. Examination reveals decreased breath sounds but no hyperresonance. An emergent chest radiograph is relatively normal except for some mild atelectasis. Which of the following is the most likely diagnosis? Acute bronchitis Pleural effusion Pneumothorax Pulmonary embolism

Correct Answer ( D ) Explanation: Pulmonary embolism refers to the obstruction of a pulmonary artery by thrombus, tumor, air or fat that originated elsewhere in the body, mostly from the deep veins of the lower extremities. It is classified as acute or chronic, and massive or submassive. It is often a fatal disease, leading to a mortality rate of 30% without treatment. Risk factors include immobilization, surgery or central venous instrumentation within the last three months, stroke/paresis/paralysis, cancer, chronic cardiac disease, autoimmune disease, obesity, >1 pack per day tobacco use, hypertension and a history of deep vein thrombosis. Most patients experience dyspnea with or without wheezing, cough, pleuritic chest pain, orthopnea, lower extremity pain or swelling, tachypnea, tachycardia, jugular venous distension, decreased breath sounds and an accentuated pulmonic component of S2. These signs and symptoms are variable and nonspecific. Furthermore, up to 32% of patients present asymptomatically. As such, the diagnosis can be difficult. The mainstay of treatment is anticoagulation.

A 38-year-old man presents with fever, fatigue, cough, and increasing dyspnea. The patient denies chills or night sweats. On physical exam, lungs are clear to auscultation bilaterally. Hepatosplenomegaly is noted on abdominal exam. Chest X-ray findings include bilateral hilar adenopathy and diffuse reticular infiltrates. Labs reveal leukocytopenia, hypercalcemia, and elevated erythrocyte sedimentation rate. What is the most likely diagnosis in this patient? Histoplasmosis Mycoplasma pneumoniae Pulmonary tuberculosis Sarcoidosis

Correct Answer ( D ) Explanation: Sarcoidosis is a multi-organ disease, usually of idiopathic origin. It is more common in the African American population and the European caucasian population. Patients commonly present with respiratory symptoms, including cough, dyspnea, and chest discomfort. Patients may also have malaise, fever, and multiple non-pulmonary symptoms. Sarcoidosis is characterized by noncaseating granulomatous inflammation in multiple affected organs. Histoplasmosis (A) patients are often asymptomatic. Chest X-ray shows a chronic fibrocavitary pneumonia. Mycoplasma pneumoniae (B) is associated with a gradual increase in constitutional symptoms and non-productive cough. Chest X-ray shows patchy infiltrates. Pulmonary tuberculosis (C) is associated with cough, night sweats and chills. Chest X-ray shows apical cavitation.

Which of the following can cause an exudative pleural effusion? Cirrhosis Congestive heart failure Nephrotic syndrome Systemic lupus erythematosus

Correct Answer ( D ) Explanation: Systemic lupus erythematosus can cause an exudative pleural effusion. A pleural effusion is an accumulation of fluid in the pleural space and can either be exudative or transudative. Exudates result primarily from pleural and lung inflammation which result in increased capillary and pleural membrane permeability. It can also be caused by impaired lymphatic drainage of the pleural space. Disease in virtually any organ can cause exudative pleural effusions by a variety of mechanisms, including infection, malignancy, immunologic responses, lymphatic abnormalities, noninfectious inflammation, iatrogenic causes, and movement of fluid from below the diaphragm. Some examples include pneumonia, tuberculosis, pancreatitis, connective tissue disorders such as lupus or rheumatoid arthritis, and malignancy, more commonly breast, lung, lymphoma or leukemia.

A 25-year-old man presents for evaluation of fever and cough. He reports last week that he was diagnosed with influenza. In the last 2 days he developed a worsening cough productive of large amounts of sputum. Vital signs are T 101°F, HR 98, BP 120/60, RR 18, and 95% oxygen saturation on room air. His chest X-ray demonstrates a lobar infiltrate in the left lower lobe. Which of the following would you most likely expect to see on the patient's Gram stain? Gram negative bacilli Gram negative diplococci Gram positive bacilli Gram positive cocci in clusters

Correct Answer ( D ) Explanation: The patient had a recent influenza infection and now presents with a lobar infiltrate. Staphylococcus aureus pneumonia is classically associated with causing post-influenza bacterial pneumonia. On Gram stain this is seen as Gram positive cocci in clusters

A 67-year-old woman presents with shortness of breath and a cough for 3 days. The patient admits to recently having a fever, chills, rhinorrhea, and myalgias for 10 days. She had one episode of diarrhea and has been nauseated. Her past medical history is positive for COPD. She does not smoke but she drinks two cans of beer daily. In the ED, her vital signs are BP 120/76, HR 108, RR 20, oxygen saturation 97% on room air, and temperature 101.2°F. A chest X-ray shows a left lower lobe consolidation. It is suspected that this patient has pneumonia caused by Staphylococcus aureus. Which aspect of this patient's history supports this suspicion? Alcohol consumption Gastrointestinal symptoms History of COPD Recent viral symptoms

Correct Answer ( D ) Explanation: The patient's recent history of fever, chills, rhinorrhea, and myalgias raises suspicion for influenza. Staphylococcus aureus is a common pathogen causing pneumonia in those currently or recently infected with influenza virus. Patients with pneumonia often present with tachypnea, tachycardia, dyspnea, chest pain, cough, fevers, and rales. S. aureus, a common nosocomial pathogen in adults, can lead to a necrotizing pneumonia. In those with gastrointestinal (B) or neurologic symptoms, Legionella pneumophilia should be considered. In those with excessive alcohol consumption (A), a common pathogen is Klebsiella pneumoniae. A history of COPD (C) raises suspicion for Haemophilius influenzae and Pseudomonas aeruginosa.

A full term male infant is delivered by cesarean section because of dystocia due to macrosomia. Apgar scores are 8 and 10. An hour after delivery he begins to have tachypnea without hypoxemia. A chest radiograph shows diffuse parenchymal infiltrates and fluid in the pulmonary fissures. The symptoms resolve without treatment within 24 hours. What is the most likely diagnosis? Laryngomalacia Meconium aspiration syndrome Respiratory distress syndrome Transient tachypnea of the newborn

Correct Answer ( D ) Explanation: This child had transient tachypnea of the newborn, the most common cause of neonatal respiratory distress. It is a benign condition due to residual pulmonary fluid remaining in the lungs after delivery. Signs of respiratory distress such as tachypnea, nasal flaring, grunting, retractions, hypoxia, and increased oxygen requirement become evident shortly after birth. The disorder is transient, with symptoms usually resolving within 72 hours after birth. Risk factors include cesarean delivery, macrosomia, male gender, and maternal asthma and diabetes mellitus. Chest radiography is the diagnostic standard for transient tachypnea of the newborn. The characteristic findings include prominent perihilar streaking, which correlates with the engorgement of the lymphatic system with retained lung fluid, and fluid in the fissures. Small pleural effusions may be seen. Patchy infiltrates have also been described. Medical care of transient tachypnea of the newborn is supportive. As the retained lung fluid is absorbed by the infant's lymphatic system, the pulmonary status improves.

A 29-year-old woman presents to the ED complaining of pain on inspiration. Over the previous 3 days, she has experienced a low-grade fever, sore throat, and body aches. Auscultation of her lungs reveals normal breath sounds. Her upright chest radiograph is seen above. Which of the following is the most appropriate next step in management? D-dimer Doxycycline Inspiratory and expiratory radiographic views Supportive care

Correct Answer ( D ) Explanation: This patient has pleuritic chest pain in the setting of a recent upper respiratory infection (URI). Viral pleuritis and pulmonary infarction are commonly associated with pleuritic chest pain. The patient's history often helps to establish the diagnosis of pleural inflammation. The pain of viral pleuritis usually is preceded by several days of a typical viral prodrome, with low-grade fever, sore throat, and other upper respiratory or constitutional symptoms. Management of viral pleuritis includes supportive care with NSAIDs. In the absence of these prodromal symptoms, an alternative etiology for pleuritis should be sought. A patient history of congestive heart failure, liver disease, uremia, or malignancy should direct further evaluation.

A 26-year-old man presents with a 2-week history of fever and a cough. He was diagnosed with HIV four months ago and is not on any antiretroviral medications. His vital signs are BP 122/76, HR 78, RR 16, oxygen saturation 92% on room air, and temperature 99.2°F. Chest X-ray demonstrates diffuse interstitial infiltrates bilaterally. Given this presentation, it is suspected that he has pneumonia secondary to Pneumocystitis jiroveci. Which of the following antibiotics is used to treat suspected pneumonia caused by Pneumocystitis jiroveci? Azithromycin Clindamycin Penicillin Trimethoprim-sulfamethoxazole

Correct Answer ( D ) Explanation: This patient has pneumonia secondary to Pneumocystitis jiroveci (PCP pneumonia). PCP generally occurs in immunocompromised patients, including HIV patients with a CD4 count less than 200 cells/mL. Once a patient's CD4 count is less than 200 cells/mL, daily prophylaxis should be initiated with trimethoprim-sulfamethoxazole to prevent PCP. Patients with PCP often present with a cough, unexplained fever, shortness of breath, and hypoxia out of proportion to the clinical picture. Chest X-ray classically shows diffuse interstitial infiltrates, but a negative chest X-ray can be seen in up to 20% of patients. Since the organism cannot be grown in the laboratory, diagnosis depends on other means. Most often, diagnosis is confirmed through bronchoscopy with a bronchoalvelolar lavage. A classic finding in patients with PCP is an elevated lactate dehydrogenase. Treatment, which should be initiated upon early suspicion and before diagnosis, is with trimethoprim-sulfamethoxazole. In addition, patients who are hypoxic with a PaO2 of less than 70 mm Hg (equivalent to an oxygen saturation of <~93%) or have an elevated alveolar-arterial gradient of greater than 35 mm Hg, should receive steroids prior to initiation of antibiotics. Azithromycin (A) is a macrolide antibiotic commonly prescribed for community-acquired pneumonia. Clindamycin (B) is generally given for aspiration pneumonia. Penicillin (C) is infrequently prescribed due to high resistance, but it is remains the first line treatment for syphilis.

A 35-year-old woman comes to the urgent care clinic complaining of a 3-week history of difficulty seeing, blurred vision, eye pain, and cough. She describes the cough as being dry and nonproductive. She has no past medical history and takes no medications. Her blood pressure is 140/85 mm Hg and her temperature is 38.1°C (100.5°F). Ophthalmologic examination shows uveitis. Chest X-ray shows bilateral hilar adenopathy. Which of the following additional findings would also most likely be found in this patient? Ferruginous bodies Increased sweat chloride Positive PPD skin test Subcutaneous nodules

Correct Answer ( D ) Explanation: Uveitis, bilateral hilar adenopathy, and a dry cough most likely indicate sarcoidosis. Sarcoidosis is granulomatous disease that can form nodules in multiple organs. When these nodules form in the skin, they are referred to as subcutaneous nodules (ie, erythema nodosum). Patients with sarcoidosis typically present with fatigue, weight loss, arthritis, dry eyes, blurry vision, and respiratory symptoms (eg, cough, dyspnea). Management usually involves the use of corticosteroids (eg, prednisone).

Which of the following is the initial treatment of choice in an overweight patient with moderate obstructive sleep apnea? Albuterol nebulizer 30 minutes prior to sleep Oral appliances Tracheostomy Weight loss and continuous positive airway pressure

Correct Answer ( D ) Explanation: Weight loss and continue positive airway pressure remains the most proven and effective therapy for obstructive sleep apnea. The continuous positive airway pressure acts as a pneumatic splint, holding the airway open. In addition, even a small reduction in body weight is associated with clinically significant improvements, and weight loss alone may be curative in some patients. Other lifestyle or conservative measures may include curbing alcohol intake before bedtime and avoiding a supine posture if obstructive sleep apnea is position dependent

A 46-year-old man presents with right eye pain and a change in his vision. He was recently diagnosed with tuberculosis and started on a 4-drug regimen. Based on his history and physical exam findings, you are concerned for optic neuritis. Which of the following medications is likely responsible for his symptoms? AEthambutolCorrect Answer BIsoniazid CPyrazinamide DRifampin

Ethambutol can cause optic neuritis resulting in decreased visual acuity and possible blindness. The medication should be discontinued immediately at the first sign of visual loss. Because of the difficulty in monitoring vision in children, ethambutol is not recommended in this population.

One Step Further Question: Is the lactate dehydrogenase level high or low in an exudate pleural fluid?

High.

Rapid Review Pneumocystis Pneumonia (PCP)

Pneumocystis Pneumonia (PCP) Patient with a history of HIV Complaining of gradual onset of non-productive cough Labs will show CD4 < 200, increased LDH CXR will show bilateral infiltrates (bat wing pattern) Most commonly caused by Pneumocystis jirovecii Treatment is TMP-SMX

One Step Further Question: What should be co-administered with isoniazid?

Pyridoxine (vitamin B6) to prevent peripheral neuropathy. Latent Tuberculosis Positive PPD criteria: 15 mm: no ↑ risk 10 mm: high-risk, homeless, health-care workers, IVDU, foreign born 5 mm: immunosuppressed, recent TB contact, abnormal CXR, steroid use ​Rx: INH for 9 months

A 16-year old boy with a history of asthma presents to the ED with severe shortness of breath and audible wheezing. He uses an inhaled corticosteroid and a long acting beta-2-agonist at home daily. However, he has had to use his short acting beta-2-agonist roughly every hour for the past day. In the ED, his vital signs are BP 114/72, HR 106, RR 28, oxygen saturation 94% on room air, and temperature 99.0°F. Diffuse wheezing is appreciated and intercostal retractions are observed. After oxygen is initiated, what is the next step in management for this patient? Begin nebulized albuterol Give epinephrine Give systemic corticosteroids Prepare for intubation

The first step in treatment of an acute asthma exacerbation includes oxygen and beta-2-agonist nebulizers, such as albuterol. Ipratropium is often added to the first three doses of albuterol in those with a severe exacerbation. Response to treatment is monitored by clinical appearance, physical examination, and peak-flow measurements. A mnemonic for the ED treatment of asthma is: BIOMES - Beta-agonists, ipratropium, oxygen, magnesium sulfate, epinephrine, and steroids.

Question: Carcinoid tumors are most likely to develop in which other organ system besides the pulmonary system?

These tumors tend to develop in the gastrointestinal tract and produce similar symptoms of carcinoid syndrome.

Transfusion Complications

Transfusion Complications Massive transfusion: coagulopathy, hypothermia, hypocalcemia Febrile reaction: most common complication, fever/chills, Hemolytic reaction: ABO incompatibility, immediate fever/chills, HA Rx: stop transfusion, IVF, diuretics Allergic reaction: urticaria or hives TRALI: like ARDS Rx: stop transfusion Delayed reaction: 3-4 weeks after transfusion, decreased Hb, increased Br GVHD: immunocompromise, rash, pancytopenia, increased LFTs Prevention: irradiated blood products in immunocompromised


Ensembles d'études connexes

Measuring the force of earthquakes

View Set

(A2.6) - Dar las gracias y responder " de nada"

View Set

GCSE/A Level Computing Hexadecimal Numbers

View Set

Quiz #5. - Ch. 4 (Repetition) - [C Programming I]

View Set

HW: electric charges and coulomb's law

View Set

placenta, umbilical cord, amniotic fluid and membrane pathology

View Set

Chapter 12: Nursing Management During Pregnancy

View Set

Philosophy (THE HUMAN PERSON IN SOCIETY)

View Set